WWW.DISS.SELUK.RU

БЕСПЛАТНАЯ ЭЛЕКТРОННАЯ БИБЛИОТЕКА
(Авторефераты, диссертации, методички, учебные программы, монографии)

 

Pages:     || 2 |

«М. В. Семенов Ю. В. Старокуров А. А. Якута Москва Физический факультет МГУ 2007 ББК 22.3я721+74.262.22 Учебное издание М. В. Семёнов, Ю. В. Старокуров, А. А. Якута Методические рекомендации по подготовке учащихся к ...»

-- [ Страница 1 ] --

Методические рекомендации

по подготовке учащихся

к участию в олимпиадах

высокого уровня по физике

М. В. Семенов

Ю. В. Старокуров

А. А. Якута

Москва

Физический факультет МГУ

2007

ББК 22.3я721+74.262.22

Учебное издание

М. В. Семёнов, Ю. В. Старокуров, А. А. Якута

Методические рекомендации по подготовке учащихся к участию в олимпиадах высокого уровня по физике. М.: Физический факультет МГУ, 2007. 60 с.: ил.

ISBN 978–5–8279–0070–2 В пособии содержатся рекомендации по подготовке учащихся 8–х 11–х классов к участию в олимпиадах высокого уровня по физике. Дано описание системы олимпиад школьников по физике в РФ. Сделан краткий обзор способов подготовки учащихся к олимпиадам. Сформулированы рекомендации по подготовке школьников к участию в олимпиадах по физике. Приведены примеры условий задач теоретических и экспериментальных туров различных этапов Всероссийской олимпиады школьников по физике третьего (Московского регионального), четвертого (федерального окружного) и пятого (заключительного), а также примеры заданий Международной физической олимпиады. Часть задач снабжены решениями и ответами.

В пособии помещена программа Всероссийской олимпиады школьников по физике, а также список литературы, рекомендуемой учащимся для подготовки к физическим олимпиадам.

Для школьников 8–х 11–х классов, родителей старшеклассников, школьных учителей, руководителей школьных физических кружков и всех интересующихся проблемами подготовки учащихся к участию в олимпиадах высокого уровня по физике.

ББК 22.3я721+74.262. Электронная версия http://genphys.phys.msu.ru/ol/ Методические рекомендации по подготовке учащихся к участию в олимпиадах высокого уровня по физике Иллюстрации Старокуров Юрий Владимирович Подп. к печати 13.09.2007.

Формат 6090 1 /16. Печать офсетная. Объём 5 печ. л.

Заказ 119. Тираж 100 экз.

Отдел оперативной печати физического факультета МГУ им. М.В. Ломоносова.

119991, Москва, ГСП–1, Ленинские горы, дом 1, строение 2.

Телефон: (495) 939–31–60.

978–5–8279–0070– 1. Система олимпиад школьников по физике в РФ Всероссийская олимпиада школьников по физике проводится во всех регионах России для учеников 9–х, 10–х и 11–х классов (в некоторых регионах, в том числе в г. Москве, олимпиада проводится также и для учеников 7–х и 8–х классов). Порядок проведения олимпиады установлен Положением о Всероссийской олимпиаде школьников (утверждено приказом Минобразования России от 30.10.2003 №4072). В соответствии с этим Положением, Всероссийская олимпиада включает в себя пять этапов: школьный, районный, региональный, федеральный окружной, заключительный. Школьный этап (I этап) проводится общеобразовательными учреждениями в октябре. Районный этап (II этап) проводится органами местного самоуправления или местными органами управления образованием в ноябре. Региональный этап (III этап) проводится в субъектах Российской Федерации совместно государственными органами управления образованием и советами ректоров высших учебных заведений в январе. Федеральный окружной этап (IV этап) проводится по семи округам: Южный, Уральский, Центральный, Приволжский, Сибирский, Северо-Западный, Дальневосточный в марте. Городские олимпиады г. Москвы и г. Санкт-Петербурга приравниваются по статусу к четвертому (федеральному окружному) этапу олимпиады.

Заключительный этап (V этап) проводится Министерством образования и науки в апреле. В настоящем пособии речь пойдет о подготовке учащихся школ г. Москвы к участию в третьем, четвертом и пятом этапах Всероссийской олимпиады школьников по физике (три перечисленных этапа часто обозначают общим термином олимпиады высокого уровня ).

III этап Всероссийской олимпиады, проходящий в г. Москве, называется окружным этапом Московской региональной олимпиады школьников (не следует путать его с федеральным окружным), поскольку проходит в десяти административных округах г. Москвы.

Третий и четвертый этапы Всероссийской олимпиады в Москве проводятся в соответствии с Положением о московской региональной олимпиаде школьников (утверждено Приказом Департамента образования г. Москвы от 26.12.2003 №1083). Третий этап организовывается Московским институтом открытого образования (МИОО), методическими центрами окружных управлений образования административных округов г. Москвы и рядом московских вузов. В последние годы в проведении окружного этапа для учащихся 11 классов участвовали физический факультет МГУ им. М. В. Ломоносова, Московский государственный технический университет имени Н. Э. Баумана, Российский университет дружбы народов, Московский институт радиоэлектроники и автоматики, Московский государственный технологический университет СТАНКИН, Московский государственный институт электронной техники, Московский государственный горный университет, Российский государственный технологический университет (МАТИ), Московский педагогический государственный университет, Московский государственный университет природообустройства, Московский государственный университет геодезии и картографии, Московский государственный строительный университет.

Окружной этап олимпиады по физике в г. Москве проходит в начале февраля. В нем могут принимать участие все желающие ученики 7–х 11–х классов независимо от места обучения и места проживания. Задания для окружного этапа готовятся кафедрой физики МИОО.



Учащимся 7–х, 8–х и 9–х классов предлагаются по 4 задачи, учащимся 10–х и 11–х классов по 5 задач. На решение одной задачи отводится астрономический час (60 минут). Окружной этап для учащихся 7–х 10–х классов проходит, как правило, в гимназиях и лицеях административных округов г. Москвы, определяемых окружными методическими центрами, а для учащихся 11–х классов в московских вузах, подавших заявку на участие в проведении олимпиады. Количество победителей (награжденных дипломом 1 степени) окружного этапа не может превышать 5% от общего количества участников олимпиады, а количество призеров (награжденных дипломами 2 и 3 степеней) не должно превышать 20% от общего числа участников олимпиады. Победители и призеры этого этапа получают дипломы окружных управлений образования; при этом победители и призеры, обучающиеся в 11–м классе, пользуются льготами при поступлении в ряд московских вузов. Государственные вузы, находящиеся на территории г. Москвы, имеют право принимать победителей и призеров окружного этапа олимпиады вне конкурса или засчитывать результаты их выступления на олимпиаде в качестве соответствующего вступительного экзамена. Примеры задач окружного этапа и их решения приведены в п. 4 и 6 пособия.

IV этап Всероссийской олимпиады, проходящий в г. Москве, называется городским этапом Московской региональной олимпиады школьников по физике (или просто городской олимпиадой по физике ). Он организовывается Московским институтом открытого образования (МИОО) и физическим факультетом МГУ им. М. В. Ломоносова.

Задания для городского этапа разрабатываются городской методической комиссией по физике. Городской этап проходит в феврале–марте и включает в себя три тура: два теоретических и экспериментальный.

В первом теоретическом туре городского этапа, который проходит на физическом факультете МГУ им. М. В. Ломоносова, могут принимать участие все желающие ученики 7–х 11–х классов независимо от места обучения, места проживания и от результатов участия в окружном этапе олимпиады. Учащимся 7–х и 9–х классов на теоретических турах городского этапа предлагаются по 4 задачи, учащимся 8–х классов задачи, а учащимся 10–х и 11–х классов по 5 задач. На решение одной задачи отводится 1 астрономический час. В 7 классе победители и призеры городского этапа определяются по итогам первого теоретического тура.

Для участия во втором теоретическом туре городского этапа, который проходит через одну–две недели после первого теоретического тура, приглашаются ученики 8–х 11-х классов, показавшие лучшие результаты в первом туре. Во второй тур традиционно проходят 30–40% участников первого тура. Количество задач и время, отводимое на их решение, во втором туре такое же, как и в первом. По итогам второго теоретического тура определяются победители и призеры городского этапа олимпиады в старших классах. В соответствии с Положением о московской региональной олимпиаде школьников, количество победителей городского этапа должно быть не более 10 человек и не более 5% от числа участников первого теоретического тура, а количество призеров должно быть не более 150 человек и не более 20% от числа участников первого теоретического тура. Победители и призеры городского этапа получают дипломы Департамента образования г. Москвы; при этом победители и призеры, обучающиеся в 11–м классе, пользуются льготами при поступлении во многие московские вузы. Для государственных вузов, находящихся на территории г. Москвы, по решению Ученого Совета вуза, согласованному с Советом ректоров и Департаментом образования, победители и призеры городского этапа Олимпиады принимаются в вуз без экзаменов или результаты их выступления на Олимпиаде зачитываются в качестве соответствующего экзамена;

при этом диплому 1 степени соответствует наилучший из возможных результатов экзамена. Примеры задач теоретических туров городского этапа и их решения приведены в п. 4 и 6 пособия.

Третий (экспериментальный) тур городского этапа является отборочным. Его итоги учитываются при формировании состава сборной команды г. Москвы для участия в заключительном этапе Всероссийской олимпиады школьников по физике. Третий тур проходит в МИОО в начале марта, на него приглашаются по 14–16 учеников из 9–го, 10–го и 11–го классов школ г. Москвы, показавшие лучшие результаты среди победителей и призеров городского этапа. На экспериментальный тур также приглашаются победители и призеры заключительного этапа Всероссийской олимпиады прошлого года, имеющие персональные приглашения на участие в очередном заключительном этапе Всероссийской олимпиады. На экспериментальном туре учащимся предлагается выполнить по 2 экспериментальные задачи, на каждую из которых отводятся 2 астрономических часа. Примеры задач экспериментального тура городского этапа приведены в п. 5 пособия.

Все участники экспериментального тура городского этапа становятся кандидатами на включение в сборную команду г. Москвы для участия в заключительном этапе Всероссийской олимпиады. Для подготовки к заключительному этапу олимпиады проводятся специальные сборы. Они проходят с середины марта до середины апреля на базе кафедры физики МИОО. В ходе сборов опытные преподаватели консультируют кандидатов в сборную г. Москвы, учат их решать различные типы теоретических и экспериментальных олимпиадных задач.

Результаты выступления учащихся на втором и третьем турах городского этапа составляют основу рейтинга, который продолжает вестись и во время сборов. Важным этапом в подготовке сборной команды является контрольная работа по решению задач федерального окружного этапа олимпиады, которые предлагаются участникам в семи федеральных округах Российской Федерации. Эти задачи разрабатываются методической комиссией Министерства образования и науки РФ; по стилю они несколько отличаются от задач, предлагаемых на московских физических олимпиадах. В дальнейшем, на заключительном этапе, московские школьники соревнуются именно с теми учащимися из других регионов РФ, которые наиболее успешно справились с задачами федерального окружного этапа. Примеры таких задач и ответы к ним приведены в п.

4 и 6 пособия. Информацию об окружном и городском этапах Московской региональной олимпиады школьников по физике можно получить в сети Internet по адресу http://genphys.phys.msu.ru/ol/.

V (заключительный) этап Всероссийской олимпиады школьников проходит в апреле, каждый год в разных регионах России (в соответствии с приказами Министерства образования и науки РФ). В команду г. Москвы, как правило, включаются 7 учащихся 9 класса и по 5 учащихся 10 и 11 классов. Кроме того, в состав команды входят школьники, имеющие персональные приглашения на участие в заключительном этапе Всероссийской олимпиады, как победители V этапа олимпиады прошлого года. Таким образом, в московскую команду обычно входят 22–25 школьников. Заключительный этап Всероссийской олимпиады включает в себя теоретический и экспериментальный туры. На теоретическом туре учащимся 9–х классов предлагается решить 4 задачи, а учащимся 10–х и 11–х классов по 5 задач. На экспериментальном туре все учащиеся выполняют по 2 задачи. На решение каждой теоретической задачи дается 1 астрономический час, а на выполнение каждой экспериментальной задачи 2 астрономических часа. Победителями и призерами заключительного этапа становятся до 40% от общего числа его участников. Победители и, как правило, все призеры в 9–м и 10–м классах получают персональные приглашения на участие в заключительном этапе Всероссийской олимпиады следующего года.

Победители и призеры, обучающиеся в 11–м классе, получают право на зачисление без экзаменов в высшие учебные заведения РФ для обучения по физико-математическим и многим техническим специальностям.

Примеры задач теоретического и экспериментального туров заключительного этапа приведены в п. 4 и 5 пособия, ответы к задачам теоретического тура заключительного этапа в п. 6 пособия.

Победители заключительного этапа Всероссийской олимпиады в 10–м классе, получившие дипломы 1 и 2 степеней (около 20 человек), становятся кандидатами в члены сборной команды РФ для участия в Международной физической олимпиаде следующего года. Они приглашаются на специальные тренировочные сборы в Московском физикотехническом институте, на которых их знакомят со спецификой задач международных олимпиад и готовят к участию в заключительном этапе Всероссийской олимпиады следующего года. По итогам этого заключительного этапа 8 лучших одиннадцатиклассников из числа кандидатов входят в сборную команду РФ, из них пятеро участвуют в Международной олимпиаде. Примеры условий задач Международных физических олимпиад приведены в п. 4 пособия.

2. Особенности олимпиадных задач, общий обзор способов подготовки учащихся к олимпиадам Задачи, которые предлагаются участникам олимпиад высокого уровня, несколько отличаются от типовых школьных задач. Главная характерная особенность олимпиадной задачи ее нестандартность, то есть внешняя непохожесть на типовые задачи. Для решения большинства олимпиадных задач практически никогда не требуется знание материала, изучение которого не предусмотрено школьными программами физики и математики. Однако, решение олимпиадных физических задач требует умения строить физические модели, глубокого понимания физических законов, умения самостоятельно применять их в различных ситуациях, а также свободного владения математическим аппаратом (без последнего получение решения большинства физических задач невозможно).

Трудность олимпиадных задач естественным образом возрастает с каждым следующим этапом олимпиады. Задачи окружного этапа лишь немного сложнее типовых школьных задач. Решение таких задач, как правило, не должно представлять трудности для школьника, который успешно освоил соответствующие разделы школьного курса физики. Поэтому неудачное выступление учащегося на окружном этапе олимпиады свидетельствует о том, что для начала нужно сосредоточить внимание на более глубоком изучении основных вопросов школьного курса физики.

Задачи теоретических туров городского этапа существенно более сложны. Решение многих из них часто требует знания приемов и методов, которые не выходят за рамки школьной программы, но, как правило, специально в школе не изучаются, либо изучаются недостаточно хорошо. (В качестве одного из примеров таких приемов можно привести методы расчета сопротивлений электрических цепей постоянного тока, состоящих из бесконечного числа резисторов.) Поэтому для успешного выступления на городском этапе олимпиады необходима специальная подготовка: желательно, чтобы учащийся посещал какие-либо дополнительные занятия по физике, либо самостоятельно работал со специальными пособиями, в которых рассматриваются методы решения олимпиадных физических задач. Список таких пособий, которые могут быть полезны при подготовке к олимпиадам, приведен в п. 8 пособия. Кроме того, необходимо пытаться самостоятельно решать физические задачи, более сложные, чем стандартные школьные. Также очень полезно познакомиться с публикациями в журнале Квант, в особенности со статьями и задачами, опубликованными в рубриках Задачник ”Кванта”, Физический факультатив, Практикум абитуриента, Варианты вступительных испытаний и Олимпиады (с материалами, опубликованными в этом журнале в прошлые годы, можно ознакомиться в сети Internet по адресу kvant.mccme.ru). В п. 3 пособия даны некоторые рекомендации для учащихся, которые желают подготовиться к участию в олимпиадах школьников по физике, а также для их родителей и учителей.

Задачи теоретического тура заключительного этапа Всероссийской олимпиады идейно во многом схожи с задачами городского этапа, но часто более сложны технически и содержат не один, а несколько разных вопросов. Опыт показывает, что учащиеся, занявшие призовые места на городском этапе олимпиады (что возможно только при условии предварительной подготовки) и специально готовящиеся к заключительному этапу, как правило, успешно выступают на нем. Специальная подготовка к заключительному этапу состоит в посещении специальных сборов команды и в решении задач заключительных этапов Всероссийских олимпиад прошлых лет.

Задачи Международной физической олимпиады значительно превосходят по трудности задачи Всероссийских олимпиад. Многие из них требуют для своего решения знаний, значительно выходящих за рамки школьных программ физики и математики. Уровень знаний учащихся, готовых к выступлению на Международной олимпиаде, больше соответствует уровню студентов младших курсов хорошего физико-математического вуза. Кроме того, для успешного выступления на Международной олимпиаде необходимо быть знакомым с особой спецификой заданий, предлагаемых на этих олимпиадах, а также с правилами оформления работ учащихся и с критериями их оценки.

Поэтому эффективная подготовка к участию в Международной физической олимпиаде возможна только с помощью квалифицированных тренеров, которые работают с кандидатами в сборную команду РФ во время специальных сборов.

Отдельно следует остановиться на физических задачах экспериментальных туров, предлагающихся на четвертом и пятом этапах Всероссийской олимпиады. Простейшим вариантом такой задачи является школьная лабораторная работа по физике. При выполнении такой работы необходимо, используя данное оборудование, измерить какиелибо физические величины. Основное отличие школьной лабораторной работы от экспериментальной олимпиадной задачи состоит в следующем. При выполнении лабораторной работы учащийся может воспользоваться учебником, в котором описаны методика измерений (указано, что и как нужно делать) и способ обработки получающихся экспериментальных данных (приведены формулы для расчетов). При решении олимпиадной экспериментальной задачи учащийся должен сам придумать, как ему провести измерения, выбрать наилучшую методику, а затем самостоятельно обработать полученные экспериментальные данные и оценить точность полученных результатов. В связи с тем, что при обучении физике в школе практически не уделяется внимание развитию экспериментальных навыков учащихся, решение экспериментальных задач традиционно вызывает значительные затруднения у школьников. Поскольку экспериментальные задачи являются обязательной составляющей заданий физических олимпиад высокого уровня, то подготовка к решению таких задач одна из необходимых составляющих подготовки учащихся к олимпиадам. Для такой подготовки желательно посещать специальные занятия (например, физические кружки), которые ведут опытные преподаватели, знакомые со спецификой экспериментальных задач. Но можно (и нужно) учиться решать такие задачи самостоятельно. Условия экспериментальных задач с перечнем необходимого оборудования и указаниями к решению опубликованы в сборниках олимпиадных заданий прошлых лет (см. п. 8 пособия). Бльшая часть необходимого экспериментального оборудования недорого стоит и может быть приобретена в радиотехнических, хозяйственных и промтоварных магазинах (многие экспериментальные задачи и вовсе можно решить, используя лишь предметы, широко применяемые в быту).

Сложные приборы (например, осциллографы), которые требуются для решения некоторых задач, как правило, имеются в школьных кабинетах физики.

В заключение этого пункта следует упомянуть о таком важном вопросе, как соотношение между школьной программой изучения физики и программой Всероссийских физических олимпиад (см. п. 7 пособия). Все задачи четвертого и пятого этапов Всероссийской олимпиады по физике составляются с учетом требований указанной программы.

Преподавание физики во многих школах г. Москвы в настоящее время ведется с использованием программ, которые не в полной мере соответствуют программе Всероссийской физической олимпиады. В качестве примера таких программ можно упомянуть следующие (в скобках указаны авторы и названия школьных учебников, по которым ведется преподавание).

Физика. 7–9 классы. (Перышкин А. В. Физика 7 кл. ; Перышкин А. В., Гутник Е. М. Физика 8 кл. ; Перышкин А. В., Гутник Е. М.

Физика 9 кл. ).

Физика. 7–11 классы. (Громов С. В., Родина Н. А Физика–7 ;

Громов С. В., Родина Н. А Физика–8 ; Громов С. В., Родина Н. А Физика–9 ; Громов С. В. Физика. Механика. 10 кл. ; Громов С. В.

Физика. Молекулярная и квантовая физика. 11 кл. ; Громов С. В.

Физика. Основы теории относительности и классической электродинамики. 10–11 кл. ).

В связи с указанным обстоятельством учащимся, которые обучаются в школе по перечисленным и некоторым другим программам, для успешного участия в физических олимпиадах высокого уровня обязательно нужно дополнительно готовиться, самостоятельно или с помощью учителей осваивать не изученные в школе разделы программы и учиться решать задачи по соответствующим темам.

Следует отметить, что в некоторых (преимущественно физикоматематических) школах города преподавание ведется по программам, предполагающим углубленное изучение физики. В качестве примера таких программ можно привести следующие.

Физика. 7–11 классы. Авторский коллектив ИОШ РАО. (Перышкин А. В., Родина Н. А. Физика–7, Физика–8 ; Лукашик В. И.

Сборник задач по физике. 7–8 класс ; Кикоин И. К., Кикоин А.К.

Физика–9 ; Мякишев Г. Я., Буховцев Б. Б. Физика–10, Физика– 11 ; Рымкевич А. П. Сборник задач по физике. 9–11 класс ; Степанова Г. Н. Сборник задач по физике. 9–11 класс ).

Физика 10–11 классы углубленного изучения. (Мякишев Г. Я. и др. Физика. Механика. 10–11 кл. ; Мякишев Г. Я. и др. Физика. Молекулярная физика. Термодинамика. 10–11 кл. ; Мякишев Г. Я. и др.

Физика. Электродинамика. 10–11 кл. ; Мякишев Г. Я. и др. Физика.

Колебания и волны. 10–11 кл. ; Мякишев Г. Я. и др. Физика. Оптика.

10–11 кл. ).

Обучение по этим программам позволяет учащимся лучше подготовиться к участию в физических олимпиадах высокого уровня, хотя необходимость в большой самостоятельной дополнительной работе учащегося все равно сохраняется. В настоящее время не существует единого учебного пособия, при помощи которого можно было бы в полной мере подготовиться к участию в физических олимпиадах высокого уровня, не прибегая к дополнительным источникам информации.

3. Рекомендации по подготовке учащихся к участию в олимпиадах высокого уровня по физике В настоящее время издано большое количество литературы, которая может быть использована для подготовки учащихся к участию в олимпиадах высокого уровня по физике (как при самостоятельных занятиях, так и при работе с учителем). Краткий список литературы, которая рекомендуется для подготовки к участию в Московской региональной олимпиаде школьников и в заключительном этапе Всероссийской олимпиады, приведен в конце данного пособия. В этих книгах можно найти большое количество задач, которые в разные годы предлагались участникам различных физических олимпиад. Многие задачи, опубликованные в этих сборниках, снабжены решениями. Дадим краткие пояснения к указанному списку литературы.

В первом издании книги [1] (1949 г.) приведены задачи с решениями Московских городских олимпиад школьников по физике 1939, 1940, 1941, 1944–1948 годов. В последующих изданиях данной книги некоторые из задач были исключены. Материалы Московских городских олимпиад по физике разных лет частично содержатся в книгах [2], [3] и [4]. В книге [5], вышедшей в серии Библиотечка ”Квант”, опубликованы задачи Московских городских олимпиад по физике 1968–1985 годов.

В недавно вышедшей книге [6] содержатся условия 475 задач, которые предлагались ученикам 8–х 11–х классов на теоретических турах Московских городских олимпиад по физике в 1986–2005 гг. Все задачи снабжены подробными решениями. Условия и решения задач Московской городской олимпиады школьников по физике 2006 года можно найти в пособии [7].

К задачам олимпиадного уровня трудности можно также отнести задачи, опубликованные в пособиях и сборниках [8], [9], [10]. Особо следует отметить задачник [11], созданный на основе опыта преподавания физики старшеклассникам в Новосибирском специализированноми учебно-научном центре при НГУ. В этом задачнике собрано большое количество довольно трудных школьных задач и отсутствуют решения (есть только ответы). Самостоятельная работа с этой книгой при подготовке к олимпиадам является особенно эффективной, но она возможна только при довольно высоком исходном уровне знаний учащегося.

Материалы заключительного этапа Всероссийской олимпиады школьников по физике (в прежние годы олимпиада РСФСР, Всесоюзная или Межреспубликанская олимпиада) приведены в сборниках [12] (1965–1980 гг.) и [13] (1992–2004 гг.). В последней книге также помещены задания, предлагавшиеся на IV (федеральном окружном) этапе Всероссийской олимпиады, а также задачи экспериментальных туров IV и V этапов Всероссийской олимпиады. Кроме того, в журнале Квант [21] опубликована бльшая часть материалов Всесоюзных олимпиад по физике, начиная с 1970 года.

Интересующиеся школьники могут ознакомиться с материалами Международной олимпиады по физике, которые опубликованы в изданиях [14] (задачи 1967–1984 годов) и [15] (задачи 1985–1999 годов). Задачи Международных физических олимпиад последних лет (на английском языке) можно найти в сети Internet [24].

Для подготовки к олимпиаде учащихся 7–х 8–х классов можно рекомендовать книгу [16] (следует помнить, что на момент ее издания в нашей стране было введено десятилетнее полное среднее образование, поэтому 6–й и 7–й классы того времени соответствуют нынешним 7–му и 8–му классам).

Из изданий, которые могут быть использованы при подготовке к физическим олимпиадам, следует упомянуть недавно вышедшую в свет книгу [17], в которой содержатся задачи Международной олимпиады по физике Туймаада, проводящейся в течение последних 10 лет для школьников старших классов в Якутском государственном университете. Также весьма полезным, особенно на начальном этапе подготовки к олимпиадам, является классический задачник [18].

Информацию, которая может быть полезна при подготовке к олимпиадам по физике, также можно почерпнуть в сети Internet, обратившись по адресам [19], [20], [22] и [23].

Подготовка учащегося к участию в олимпиадах по физике должна включать в себя несколько составляющих. Прежде всего, учащийся должен полно и всесторонне освоить материал школьной программы соответствующего класса по физике и математике без этого достичь высоких результатов при выступлении на физической олимпиаде невозможно. В дополнение к материалу школьной программы необходимо осваивать дополнительные разделы школьного курса физики в объеме, предусмотренном программой Всероссийских физических олимпиад (п. 7 пособия). Критерием успешности подготовки к окружному этапу олимпиады может служить способность учащегося к решению задач по соответствующим темам из задачника [18], а также из сборников [3] и [4].

При подготовке к теоретическим турам городского этапа олимпиады учащемуся необходимо разбирать задачи из сборников [6] и [5] (пытаться решать задачи, а в случае возникновения затруднений знакомиться с их решениями), а также самостоятельно решать задачи из сборника [11] и задачи федерального окружного этапа Всероссийской олимпиады по физике из книги [13]. При работе с последней книгой учащемуся следует обратить внимание на задачи экспериментальных туров и попытаться решить и самостоятельно выполнить хотя бы некоторые из них. Это послужит хорошей подготовкой к возможному участию в экспериментальном туре городского этапа олимпиады.

На данной стадии подготовки большую пользу может принести посещение специальных занятий, которые проводятся опытными преподавателями специально для школьников, желающих принимать участие в олимпиадах высокого уровня по физике. Такие занятия, организованные Департаментом образования г. Москвы для учеников 8–х 11-х классов, проходят в Московском институте открытого образования, на физическом факультете МГУ им. М. В. Ломоносова, а также в ряде школ и лицеев г. Москвы. Все занятия являются бесплатными. С их расписанием можно ознакомиться в сети Internet по адресу http://genphys.phys.msu.ru/ol/.

Следует отметить, что далеко не все учащиеся могут успешно выступать на олимпиадах высокого уровня по физике. Для того, чтобы стать победителем такой олимпиады, необходимо не только хорошо знать материал программы по физике и иметь практические навыки решения различных задач, но и уметь находить и записывать решения задач за ограниченное время, отводимое участникам олимпиады. Последнее удается учащимся не всегда, даже если их физикоматематическая подготовка является весьма хорошей. Поэтому удовлетворительным результатом подготовки учащегося к олимпиадам можно считать его победу в окружном этапе олимпиады по физике. Если учащемуся удалось пройти во второй теоретический тур городского этапа олимпиады, то этот результат следует считать хорошим. Победа же в городском этапе олимпиады является весьма высоким, отличным достижением. Задачу подготовки учащегося к участию в олимпиадах высокого уровня по физике можно считать полностью выполненной в случае, если школьник смог стать победителем или призером заключительного этапа Всероссийской олимпиады школьников по физике.

Приступая к подготовке к участию в олимпиадах высокого уровня по физике, нужно помнить о том, что олимпиада это всего лишь интеллектуальное соревнование, которое проводится с целью повышения интереса школьников к изучению предмета. Поэтому не следует расстраиваться, если учащемуся не удалось стать победителем олимпиады по физике. В любом случае подготовка к олимпиаде позволяет глубже освоить школьную программу, изучить дополнительные вопросы курса физики, научиться решать различные типы задач (в том числе, весьма трудных). В конечном итоге, все это принесет ощутимую пользу в плане получения хорошего образования и положительно скажется при сдаче выпускных экзаменов в школе и вступительных испытаний в высшее учебное заведение.

4. Примеры условий задач теоретических туров олимпиад школьников по физике Московская региональная олимпиада школьников по физике, этап административного округа г. Москвы (III этап Всероссийской олимпиады школьников) 9.1. Материальная точка движется вдоль отрезка прямой, длина этого отрезка составляет L = 2 м. Скорость точки в начале отрезка v1 = = 0,2 м/с, а в конце отрезка v2 = 0,4 м/с. Известно, что скорость точки все время увеличивалась, но ускорение не превосходило величину а0 = = 0,1 м/с2. Каким могло быть среднее ускорение точки на этом отрезке?

9.2. В системе, изображенной на рисунке, массы грузов равны 2m, m и 5m. Массы нитей и блоков пренебрежимо малы, не лежащие на блоках участки нитей вертикальны. Пренебрегая трением, найти выдернуть вилку кипятильника из сети, температура падает до +55 C за 3 минуты. Пусть при температуре +55,5 C мощность кипятильника снизилась ровно в 2 раза (например, из за падения напряжения сети).

За какое время после этого температура воды изменится на полградуса? Нагреется или охладится при этом вода? Температура в комнате +20 C.

9.4. К батарейке подключены два резистора, соединенные последовательно. Школьный вольтметр подключают к батарейке, и он показывает при этом напряжение 6 В. Напряжение, измеренное этим вольтметром на выводах первого резистора, оказалось равно 2 В, на выводах второго тоже 2 В. Отключим один из резисторов, а на его место подключим вольтметр. Что он будет показывать? Сопротивление вольтметра довольно велико, хотя идеальным его считать нельзя.

1 Материалы предоставлены А. Р. Зильберманом.

10.1. На двух сложенных вместе одинаковых нитях длиной L каждая было подвешено тело массой m. Посередине между нитями вставили очень легкую горизонтальную распорку общей длиной d, состоящую из двух соединенных торцами половинок. Найти силу, с которой одна половинка распорки действует на другую.

10.2. На гладком горизонтальном столе лежит твердый кубик.

На него налетает мягкий, но довольно упругий кубик такой же массы, и между ними происходит лобовой удар. Скорость мягкого кубика после удара уменьшилась в 5 раз. Какая часть максимальной энергии деформации перешла в тепло при этом ударе? Считайте, что тепло выделяется в мягком кубике при его деформировании.

10.3. Смесь азота и водорода при нормальных условиях имеет плотность = 0,2 кг/м3. Молярные массы азота и водорода равны µ1 = = 28 г/моль и µ2 = 2 г/моль. Найти отношение масс этих газов в смеси.

10.4. В стакан с водой опустили кипятильник. Измерения показали, что вода нагревается от +55 C до +56 C за 1 минуту. Если при температуре +56 C выдернуть вилку кипятильника из сети, то температура падает до +55 C за 2 минуты. Во сколько раз нужно снизить мощность кипятильника для того, чтобы поддерживать температуру воды равной +55 C? Хватит ли мощности этого кипятильника, чтобы довести воду в стакане до кипения? Температура в комнате +22 C.

10.5. К батарейке подключены три разных вольтметра, соединенные последовательно. Показания вольтметров составляют при этом 0,5 В, 1 В и 2 В. Изменим схему соединения подключим теперь два вольтметра параллельно, к ним присоединим последовательно третий вольтметр, а к выводам всей цепи подключим батарейку. Оказалось, что один из вольтметров при таком переключении свои показания не изменил. Что показывают остальные два вольтметра? Напряжение батарейки считать неизменным.

Московская региональная олимпиада школьников по физике, (IV этап Всероссийской олимпиады школьников) 9.1. Один корабль идет по морю на север с постоянной скоростью 20 узлов, а другой навстречу ему, на юг, с такой же скоростью. КорабИспользованы материалы издания [6].

ли проходят на очень малом расстоянии друг от друга. Шлейф дыма от первого корабля вытянулся в направлении на запад, а от второго на северо-запад (см. рисунок). Определите величину и направление скорости ветра. 1 узел = 1 морская миля в час, 1 морская миля = 1852 м.

9.2. В вертикальную стену вбиты два гвоздя так, что они лежат на одной вертикальной прямой. Кусок однородной проволоки массой m согнули в дугу в виде половины окружности и шарнирно прикрепили за один из концов к верхнему гвоздю A (см. рисунок). Дуга при этом оперлась на нижний гвоздь B. Найдите величину силы, с которой проволока давит на верхний гвоздь, если известно, что в отсутствие нижнего гвоздя, когда проволока находится в равновесии, диаметр AC дуги составляет с вертикалью угол 0. Расстояние между гвоздями равно радиусу дуги. Трения нет.

9.3. В системе, изображенной на рисунке, блоки имеют пренебрежимо малые массы, нить невесомая и нерастяжимая, не лежащие на блоках участки нити горизонтальны. Массы грузов, лежащих на горизонтальной плоскости, одинаковы и равны M. Нить тянут за свободный конец в горизонтальном направлении с силой F. С каким ускорением движется конец нити, к которому приложена эта сила? Трения нет, движение грузов считайте поступательным.

9.4. Электрическая цепь, изображенная на левом рисунке, состоит из источника постоянного напряжения U = 3 В, миллиамперметра с очень маленьким внутренним сопротивлением, четырех постоянных резисторов и одного переменного. На правом рисунке приведен график зависимости показаний миллиамперметра от величины сопротивления переменного резистора R. Найдите величины сопротивлений постоянных резисторов R1 и R2.

10.1. На массивный гладкий цилиндр радиусом R, движущийся поступательно со скоростью u, налетает маленький шарик, движущийся навстречу цилиндру перпендикулярно его оси со скоростью v (см. рисунок). Расстояние между линией, вдоль которой движется шарик, и плоскостью, в которой движется ось цилиндра, равно L (L < R). Найдите величину скорости шарика v1 после абсолютно упругого удара о цилиндр. Сила тяжести отсутствует.

10.2. Маленькую шайбу запустили по шероховатой горизонтальной поверхности со скоростью v0 = 5 м/с. График зависимости скорости шайбы v от пройденного ею пути S изображен на рисунке. Какой путь пройдет шайба до полной остановки, если ее запустить из той же точки в том же направлении со скоростью v1 = 4 м/с?

10.3. На рисунке приведен график зависимости давления насыщенного пара некоторого вещества от температуры. Определенное количество этого вещества находится в закрытом сосуде постоянного объема в равновесном состоянии, соответствующем точке A на рисунке.

До какой температуры следует охладить эту систему, чтобы половина имеющегося в сосуде вещества сконденсировалась? Объемом сконденсировавшегося вещества можно пренебречь по сравнению с объемом сосуда.

10.4. В черном ящике с двумя контактами находится схема, состоящая из незаряженного конденсатора и резистора. К контактам в момент времени t = 0 подсоединили конденсатор емкостью C, имеющий заряд Q0. График зависимости заряда на этом конденсаторе от времени изображен на рисунке. Найдите сопротивление резистора и емкость конденсатора, находящихся в черном ящике.

10.5. Рентгеновский аппарат состоит из точечных источника И и приемника П, жестко закрепленных на станине. Между источником и приемником перемещают цилиндрический толстостенный баллон (см. рисунок). При этом интенсивность A рентгеновского излучения, регистрируемого приемником, зависит от координаты x так, как показано на рисунке. Есть ли внутри баллона содержимое, поглощающее рентгеновские лучи?

Федеральный окружной (IV) этап Всероссийской олимпиады 9.1. Деревянный плот оттолкнули от берега так, что в начальный момент времени его скорость оказалась равной v0 и направленной перпендикулярно берегу (см. рисунок). Двигаясь по траектории, показанной на рисунке, плот через некоторое время T после начала движения оказался в точке A. Скорость реки постоянна и равна u. Графически найдите точки траектории плота, в которых он находился в моменты времени 2T, 3T и 4T.

9.2. Тело, движущееся по горизонтальной поверхности, за промежуток времени t1 прошло путь s1. Какой путь s2 оно может пройти за последующий промежуток времени t2 ? Коэффициент трения скольжения тела о поверхность равен µ.

9.3. В схеме, изображенной на рисунке, все вольтметры одинаковые, а их внутреннее сопротивление много больше всех остальных 3 Использованы материалы издания [13].

сопротивлений цепи. Найдите показания вольтметров, если сопротивление каждого из резисторов R = 10 Ом, а напряжение на входе цепи U = 4,5 В.

9.4. Два плоских зеркала З1 и З2, каждое из которых имеет форму квадрата со стороной a, сложены под прямым углом. Точечный источник света S располагается на расстоянии a от каждого из зеркал (схема опыта приведена на рисунке). Заштрихуйте области, находясь в которых, наблюдатель сможет увидеть ровно n изображений источника S; принять n = 0, 1, 2, 3, 4, 5.

плоскостью (см. рисунок). С каким ускорением и в какую сторону начнет двигаться этот такого радиуса r, что 100r = H?

10.2. Два точечных тела начинают одновременно двигаться: первое по окружности радиуса R с постоянной по модулю скоростью v1, второе из центра той же окружности со скоростью v2 = v1, причем вектор v2 направлен все время на первое тело (см. рисунок). На каком расстоянии друг от друга окажутся тела через время 10.3. Длинная проволока навита в спираль радиуса R с шагом h (см. рисунок); ось спирали расположена вертикально. По спирали скользит бусинка; коэффициент трения между проволокой и бусинкой равен µ. Найдите установившуюся скорость v0 движения бусинки.

10.4. В экспериментах по обнаружению нейтрино используют легкоплавкий металл галлий (tпл = 29,8 C). Прямоугольная теплоизолированная кювета шириной d, открытая сверху, до высоты Н заполнена галлием, нагретым до температуры кипения. К противоположным стенкам A и B, изготовленным из хорошо проводящего материала, подведено внешнее электрическое напряжение. Через расплав галлия начинают пропускать постоянный ток I (см. рисунок). Через какое время весь галлий выкипит? Удельную теплоту парообразования, плотность и удельное сопротивление считать известными.

10.5. В тепловой машине молей идеального одноатомного газа совершают замкнутый цикл, состоящий из процессов 1–2 и 2–3, в которых давление p газа линейно зависит от занимаемого им объема V, и изохорического процесса 3– (см. рисунок). Величины p0 и V считайте известными. Найдите:

1) температуру и давление газа в точке 3; 2) работу A, совершаемую газом за цикл; 3) коэффициент полезного действия тепловой машины.

Заключительный (V) этап Всероссийской олимпиады 9.1. С высокого берега озера за веревку подтягивают лодку. К веревке привязан флажок (см. рисунок). В момент, когда флажок оказался в точке C посередине между A и B, веревка была направлена под углом = 60 к горизонту. Найдите скорость флажка в этот момент, если известно, что скорость лодки u = 1 м/с.

4 Использованы материалы издания [13].

9.2. Горизонтальная платформа массой M = 300 г подвешена на резиновом жгуте AB (см. рисунок). Жгут проходит сквозь отверстие в грузе массой m = 100 г. Система находится в равновесии. Затем груз отпускают без начальной скорости с высоты h относительно платформы. Найдите, при каком минимальном значении h жгут порвется, если его максимально допустимое удлинение хк = 8 см. Зависимость силы натяжения жгута от его удлинения F (x) приведена на графике. Удар груза о платформу считать абсолютно неупругим.

9.3. В теплоизолированном сосуде находится смесь воды и льда при температуре t1 = 0 C. Через стенку в сосуд вводится торец медного стержня, боковые стенки которого покрыты теплоизолирующим слоем.

Другой торец стержня погружен в воду, кипящую при атмосферном давлении. Через время м = 15 мин весь лед в сосуде растаял. Если бы вместо медного стержня в этом эксперименте был использован стальной стержень того же сечения, но другой длины, то весь лед растаял бы через время с = 48 мин. Стержни соединяют последовательно (см.

рис.). Какой будет температура t в месте соприкосновения медного и стального стержней? Рассмотрите два случая: 1) кипящая вода соприкасается с торцом медного стержня; 2) кипящая вода соприкасается с торцом стального стержня. Через какое время растает весь лед при последовательном соединении стержней? Будет ли это время одинаково в случаях 1 и 2?

Найдите ее.

10.1. По двум кольцевым дорогам радиусом R, лежащим в одной плоскости, движутся автомобили A1 и A2 со скоростями v1 = v = = 20 км/ч и v2 = 2v (см. рисунок). В некоторый момент автомобили находились в точках M и C на расстоянии R/2 друг от друга. 1) Найдите скорость автомобиля A2 в системе отсчета, связанной с автомобилем A1 в этот момент. 2) Найдите скорость автомобиля A2 в системе отсчета, связанной с автомобилем A1 когда A2 окажется в точке D. Размеры автомобилей малы по сравнению с R.

10.2. В герметично закрытом сосуде находится влажный воздух, температура которого равна t1 = 75 C, а относительная влажность = 25%. Воздух в сосуде начинают охлаждать. При какой температуре t2 внутренние стенки сосуда запотеют? График зависимости давления насыщенного водяного пара (в относительных единицах) от температуры приведен на рисунке.

поглощенной газом, одинаковы. Постройте по данным задачи ось V.

вольтметром, равно E/2. 1) Определите разность потенциалов на конденсаторе C1 в этот момент. 2) Найдите силу тока через резистор R1 в этот же момент. 3) Определите максимальный заряд на конденсаторе C1. 4) Вычислите полное количество теплоты, выделившееся в цепи после замыкания ключа K.

10.5. На рисунке представлена идеализированная зависимость силы тока I, протекающего через газоразрядную трубку, от напряжения U между электродами для случая несамостоятельного газового разряда. Трубка с последовательно соединенным балластным резистором сопротивлением R = 107 Ом подключается к конденсатору емкостью С = 103 Ф, заряженному до напряжения U0 = 300 В. Какое количество теплоты выделится в трубке за время полного разряда конденсатора?

Международная физическая олимпиада Вертикально расположенный цилиндрический сосуд содержит газ под поршнем в состоянии термодинамического равновесия. Поршень представляет собой стеклянную пластинку и может свободно перемещаться. Предположим, что утечки газа нет и что трение между стеклянным поршнем и стенками цилиндра достаточное, чтобы подавлять колебания, но не вносит заметных потерь энергии. Первоначальная температура газа равна температуре окружающей среды. С хорошей точностью газ можно считать идеальным. Будем также предполагать, что стенки цилиндра (включая поршень и дно) имеют очень низкую теплопроводность и теплоемкость, поэтому теплообмен между газом и окружающей средой происходит очень медленно и им можно пренебречь.

Через стеклянную пластинку в цилиндр направляется пучок света от лазера постоянной мощности. Это излучение распространяется через воздух и стекло без поглощения, но полностью поглощается газом в сосуде. В результате молекулы газа переходят в возбужденные состояния, из которых они быстро возвращаются в основное состояние путем ступенчатого испускания инфракрасного излучения. Это излучение поглощается другими молекулами и отражается стенками сосуда и стеклянным поршнем. Таким образом, энергия лазерного излучения, поглощенная газом, трансформируется в течение очень короткого времени в энергию теплового движения молекул.

В результате стеклянный поршень смещается вверх. Спустя некоторое время лазер выключается и измеряется смещение поршня.

5 Использованы материалы издания [15].

1. Используя приведенные ниже данные, определите температуру и давление газа после облучения.

2. Вычислите механическую работу, выполненную газом в результате поглощения излучения.

3. Подсчитайте поглощенную энергию излучения.

4. Рассчитайте мощность излучения лазера, поглощенного газом, и соответствующее число поглощенных фотонов (и, следовательно, число элементарных процессов поглощения лазерных фотонов) в единицу времени.

5. Определите долю оптической энергии, преобразованной в механическую потенциальную энергию стеклянного поршня.

6. После выключения лазера цилиндр медленно поворачивают на 90 так, что его ось принимает горизонтальное положение. Теплообменом между газом и стенками сосуда по-прежнему можно пренебречь.

Изменятся ли давление и температура газа в результате такого поворота? Определите новые значения температуры и давления.

Давление окружающего воздуха р0 = 101,3 кПа.

Комнатная температура t0 = 20 C.

Внутренний диаметр цилиндра 2r = 100 мм.

Масса стеклянной пластины m = 800 г.

Количество газа в сосуде = 0,100 моль.

= 20,8 Дж/(моль · К).

Длина волны излучения лазера = 514 нм.

Время облучения t = 10,0 с.

Смещение поршня после облучения s = 30,0 мм.

Постоянная Планка h = 6,62 · 1034 Дж · с.

Газовая постоянная R = 8,314 Дж/(моль · К).

К первым успехам теории магнитных явлений Ампера относится вычисление индукции В магнитного поля, создаваемого проводником с электрическим током, и сравнение с проведенными ранее расчетами Био и Савара.

Интересным частным случаем является очень длинная проволока с постоянным током i, изогнутая в виде буквы V с углом раствора (в этой задаче углы выражаются в радианах). Согласно вычислениям Ампера, модуль магнитной индукции B в точке Р на оси V-образной проволоки на расстоянии d от вершины пропорционален tg(/2). Работа Ампера позже вошла в теорию электродинамики Максвелла и является общепризнанной.

1. Найдите направление магнитной индукции B в точке Р.

2. Зная, что модуль магнитной индукции пропорционален tg(/2), то есть B(P ) = k tg(/2), вычислите коэффициент пропорциональности k.

3. Определите модуль магнитной индукции В в точке P1, симметричной точке Р относительно вершины, т.е. на оси проволоки, и на том же расстоянии d от вершины, но внутри контура.

4. Вычислите как функцию В период малых колебаний помещенной в точку Р магнитной стрелки, если стрелка имеет момент инерции I и магнитный дипольный момент p. Стрелка колеблется в той же плоскости, в которой лежит вектор индукции B.

5. При тех же условиях, что и у Ампера, Био и Савар предполагали, что модуль магнитной индукции B в точке Р определяется выраiµ жением (используем современную запись) B(P ) = 2, где µ0 магd нитная постоянная. Они попытались с помощью эксперимента проверить справедливость соответствующих предположений (Ампера и своих), измеряя период колебаний магнитной стрелки как функцию угла. Однако для некоторых значений разница результатов столь мала, что ее трудно измерить.

Установите приближенно, какой интервал углов следует выбрать, чтобы обнаружить различие между двумя предсказаниями для периода T колебаний магнитной стрелки в точке Р, которое в их значениях составляло не менее 10%, а именно T1 > 1,10T2 (где T соответствует предположению Ампера, а T2 предположению Био и Савара).

Примечание. В зависимости от выбранного способа решения задачи может быть полезной формула Рассмотрите метод, часто применяемый для ускорения космических зондов в нужном направлении. Пролетая вблизи планеты, космический зонд может значительно увеличить свою скорость и существенно изменить направление полета за счет незначительной части энергии орбитального движения планеты. Проанализируйте случай космического зонда, пролетающего вблизи Юпитера.

Юпитер вращается вокруг Солнца по эллиптической орбите, которую можно аппроксимировать окружностью средним радиусом R.

Чтобы приступить к анализу физической проблемы, предварительно решите две задачи.

1. Определите скорость v движения Юпитера по орбите вокруг Солнца.

2. Найдите расстояние от Юпитера, на котором силы гравитационного взаимодействия зонда с Юпитером и Солнцем равны (предполагается, что зонд находится в точке на отрезке, соединяющем Солнце и Юпитер).

3. Космический зонд массой m = 825 кг пролетает вблизи Юпитера. Для упрощения предположите, что траектория зонда полностью лежит в плоскости орбиты Юпитера, т.е. можно пренебречь случаями, когда зонд покидает эту плоскость.

Рассмотрим ту область пространства, в которой притяжение Юпитера значительно превосходит все остальные гравитационные силы.

В системе отсчета, связанной с центром Солнца, начальная скорость космического зонда v0 = 1,00 · 104 м/с (в положительном направлении оси Y ), в то время как скорость Юпитера направлена в отрицательном направлении оси X (см. рисунок 1). Под начальной скоростью мы понимаем скорость космического зонда в межпланетном пространстве достаточно далеко от Юпитера, но уже в области, где притяжение Солнца пренебрежимо мало. Предположим, что взаимодействие происходит за достаточно короткий промежуток времени, так что можно пренебречь изменением направления скорости движения Юпитера по орбите. Предположим также, что зонд проходит позади Юпитера, т.е.

его координата x1 больше, чем координата x2 Юпитера в тот момент, когда координаты у равны.

Найдите направление скорости движения космического зонда (угол 0 между вектором скорости зонда и осью X и модуль его скорости v в системе отсчета, связанной с Юпитером, когда зонд находится далеко от Юпитера.

4. Найдите полную механическую энергию Е зонда в системе отсчета, связанной с Юпитером, полагая, как обычно, что потенциальная энергия на очень больших расстояниях равна нулю, а скорость зонда можно считать постоянной из-за малости всех гравитационных взаимодействий.

5. Траекторией космического зонда в системе отсчета, связанной с Юпитером, является гипербола, уравнение которой в полярных координатах имеет вид где b расстояние между асимптотой и центром Юпитера (так называемый прицельный параметр), Е полная механическая энергия зонда в системе отсчета, связанной с Юпитером, G гравитационная постоянная, М масса Юпитера, r и и полярные координаты (расстояние до центра Юпитера и угол).

На рисунке 2 показаны две ветви гиперболы, описываемой уравнением (1), и ее асимптоты в полярных координатах. Отметим, что уравнение (1) описывает гиперболу, фокус которой находится в центре притяжения, т.е. в центре Юпитера. Траектория космического зонда представляет собой ветвь притяжения и изображена на рисунке сплошной линией.

Используя уравнение (1), описывающее траекторию зонда, найдите полное угловое отклонение траектории зонда в системе отсчета, связанной с Юпитером (см. рисунок 1), и выразите его как функцию начальной скорости v и прицельного параметра b.

6. Полагая, что зонд не может пройти мимо Юпитера на расстоянии от его центра, меньшем, чем три юпитерианских радиуса, найдите минимально возможное значение прицельного параметра и максимально возможное значение углового отклонения.

7. Получите выражение для конечной скорости зонда v в системе отсчета, связанной с Солнцем, как функцию только скорости Юпитера v, начальной скорости зонда v0 и углового отклонения.

8. Используя предыдущий результат, найдите значение конечной скорости зонда v в системе отсчета, связанной с Солнцем, при максимально возможном значении углового отклонения.

Масса Солнца Мс = 1,991 · 1030 кг.

Масса Юпитера М = 1,901 · 1027 кг.

Средний радиус орбиты Юпитера R = 7,783 · 1011 м.

Экваториальный радиус Юпитера Rю = 6,98 · 107 м.

Период обращения Юпитера Тю = 374,32 Мс.

Гравитационная постоянная G = 6,673 · 1011 м3 /(кг · с2 ).

5. Примеры условий задач экспериментальных туров олимпиад школьников по физике Московская региональная олимпиада школьников по физике, (IV этап Всероссийской олимпиады школьников) 9.1. Определить плотность неизвестной жидкости.

Оборудование: весы и разновес, миллиметровая бумага, порция неизвестной жидкости в пластиковом стаканчике, металлический шарик на нити, вода в пластиковом стаканчике, салфетки.

9.2. Взвесить листок миллиметровки.

Оборудование: прямоугольный кусок миллиметровой бумаги (половина листа формата А4), старая монета достоинством 1 коп (ее масса задана в условии и составляет 1 г), карандаш только для нанесения меток!

9.3. Отградуировать неправильную пружинку.

Оборудование: специально изготовленная неправильная пружинка из двух кусков резинки разной длины, концы которых соединены (вначале растягивается только один кусок, с некоторого растяжения включается и второй, большей длины), длинная линейка или мерная лента, штатив с лапкой, гирька массой 100 г с крючком, липкая лента.

9.4. Измерить диаметр канала в иголке для шприца.

Оборудование: шприц объемом 5 мл с иголкой, вода в стакане, мерная лента, секундомер.

9.5. Измерить избыточное давление воздуха в надутом воздушном шарике.

Оборудование: надутый воздухом воздушный шарик, миллиметровая бумага, легкая книжка в жестком переплете, гирька массой 200 г, липкая лента, стаканчик с водой, салфетки.

9.6. Измерить процентное содержание льда в выданной порции снега.

6 Материалы предоставлены А. Р. Зильберманом.

Оборудование: порция рыхлого снега (с мороза или из морозильника), школьный калориметр, холодная вода, стаканчик с делениями, термометр, часы.

9.7. Измерить сопротивления нескольких резисторов.

Оборудование: резисторы с сильно различающимися неизвестными сопротивлениями (пример набора: 50 Ом, 300 Ом, 2 кОм, 5 кОм, 30 кОм), батарейка плоская (4,5 В), вольтметр школьный на 6 В, миллиамперметр школьный на 5/50 мА, провода.

10.1. Определить плотность пластмассы, из которой изготовлено тело почти правильной формы.

Оборудование: весы и разновес, миллиметровая бумага, вода в прозрачном цилиндрическом стаканчике, тело почти правильной формы костяшка домино; нитка, липкая лента.

10.2. Измерить удельную теплоемкость материала гирьки.

Оборудование: гирька, холодная вода в пластмассовом стаканчике, термометр, горячая вода по требованию, нитка, миллиметровая бумага.

10.3. Измерить отношение давлений насыщенного водяного пара при температурах +65 C и +80 C.

Оборудование: стакан для горячей воды, термометр, часы, 2 листа плотной бумаги, липкая лента, мерный стаканчик (с делениями), горячая вода по требованию.

10.4. Измерить температуру плавления и удельную теплоту плавления легкоплавкого вещества.

Оборудование: легкоплавкое вещество два кусочка массой по 3 г, стакан для горячей воды, термометр, часы, 2 листа плотной бумаги, липкая лента, мерный стаканчик (с делениями), горячая вода по требованию.

10.5. Построить вольтамперную характеристику лампочки накаливания.

Оборудование: батарейка плоская, вольтметр на 6 В, лампочка, еще две такие же лампочки, соединенные параллельно, еще три такие же лампочки, соединенные параллельно, резистор с сопротивлением 10 Ом, провода. Нужно построить зависимость тока через лампочку (в миллиамперах) от напряжения на ней (в вольтах). Три группы ламп можно использовать, как независимые элементы, но расформировывать их нельзя!

10.6. Измерить емкость необычного конденсатора.

Оборудование: батарейка плоская, конденсатор очень большой емкости (ионистор), подключенный одним выводом к батарейке (его нельзя подключать в обратной полярности!), вольтметр на 6 В, резистор с сопротивлением 1 кОм, провода.

10.7. Исследовать черный ящик из резисторов.

Оборудование: батарейка плоская, черный ящик с тремя выводами ( трехполюсник ), вольтметр на 6 В, миллиамперметр на 5 мА, провода. Нужно определить возможную схему соединений резисторов внутри ящика и величины их сопротивлений.

Заключительный (V) этап Всероссийской олимпиады 1. Определите сопротивление резистора.

Оборудование: источник тока, резистор известного сопротивления R1, резистор неизвестного сопротивления R2, стаканчик (стеклянный на 100 мл), термометр, часы (можно использовать свои наручные), миллиметровая бумага, кусок пенопласта, сосуд с водой.

2. Определите коэффициент трения бруска о стол.

Оборудование: брусок, линейка, штатив, нитки, гиря известной массы.

3. Определите плотность тела.

Оборудование: тело неправильной формы, металлический стержень, линейка, штатив, сосуд с водой, нить.

4. Исследуйте зависимость скорости струи, вытекающей из сосуда, от высоты уровня воды в этом сосуде.

Оборудование: штатив с муфтой и лапкой; стеклянная бюретка со шкалой и резиновой трубкой; пружинный зажим; винтовой зажим;

секундомер; воронка; кювета; стакан с водой; лист миллиметровой бумаги.

5. Определите температуру воды, при которой ее плотность максимальна.

Оборудование: стакан с водой при температуре t = 0 C; металлическая подставка; термометр; ложечка; часы; маленький стакан.

7 Использованы материалы издания [13].

6. Определите давление насыщенного водяного пара, находящегося при температуре t1 = 60 C, если известны атмосферное давление и давление насыщенного пара при комнатной температуре t2.

Оборудование: сосуд с горячей водой; сосуд с водой при комнатной температуре; пробирка; пробка с отверстием; термометр; линейка.

7. Найдите магнитную индукцию поля магнита на его оси на расстоянии l = 30 см от его центральной (средней) линии, если известно, что в Орле (город, где проходила данная олимпиада) магнитная индукция поля Земли B 4,1 · 105 Тл, а вектор B направлен под углом 65 к горизонту.

Оборудование: магнит в виде тонкого цилиндра длиной 40 мм;

линейка длиной 40–50 см; компас.

8. Экспериментально исследуйте зависимость удлинения мягкой пружины под действием ее собственного веса от числа витков пружины. Дайте теоретическое объяснение найденной зависимости. Определите коэффициент упругости и массу пружины. Исследуйте зависимость периода колебаний пружины от ее числа витков.

Оборудование: мягкая пружина, штатив с лапкой, рулетка, часы с секундной стрелкой, шарик из пластилина массой m = 10 г, миллиметровая бумага.

9. Определите ЭДС и внутреннее сопротивление источника тока.

Вычислите емкость двух электролитических конденсаторов.

Оборудование: источник постоянного тока, ЭДС и внутреннее сопротивление r которого неизвестны; вольтметр V с известным внутренним сопротивлением RV ; ключ; резистор с заданным сопротивлением R; два конденсатора, емкости C1 и C2 которых неизвестны; секундомер; миллиметровая бумага; полулогарифмическая бумага.

10. Найдите расстояние H между двумя тонкими стеклами, закрепленными в открытом с одной стороны ящике.

Оборудование: ящик с двумя параллельными стеклами; линейка;

карандаш; лист бумаги.

Московская региональная олимпиада школьников по физике, этап административного округа г. Москвы (III этап Всероссийской олимпиады школьников) 9.1. Скорость на отрезке за время движения возрастает на v = = 0,2 м/с. Для нахождения среднего ускорения нужно знать длительность интервала времени, за который увеличивается скорость. Это время может быть различным при разных способах разгона.

Для того, чтобы среднее ускорение оказалось максимально возможным, нужно сократить длительность интервала времени, за который увеличивается скорость, для чего нужно сделать среднюю скорость как можно большей. Для этого нужно как можно быстрее набрать скорость, очень близкую к v2 = 0,4 м/с (необходимо, чтобы дальше скорость все же увеличивалась это требуется по условию задачи). Время путешествия при этом равно времени разгона с ускорением а0 = 0,1 м/с2 плюс время прохождения остатка пути со скоростью v2 = 0,4 м/с. Время разгона составляет v/а0 = 2 секунды, за эти 2 секунды точка пройдет расстояние 0,6 м. Оставшиеся 1,4 м точка пройдет со скоростью 0,4 м/с, что потребует еще 3,5 секунды. Таким образом, общее (и минимально возможное) время движения составит t = 5,5 секунд. Следовательно, максимально возможное среднее ускорение при таких условиях разгона составит aср = v/t 0,0364 м/с2, что не превышает а0 = 0,1 м/с2, как этого требует условие задачи.

Для получения минимального среднего ускорения следует сделать время движения как можно больше, для чего нужно приберечь разгон напоследок, а предшествующий разгону участок пути пройти с минимальной скоростью. Время разгона при этом по-прежнему составит 2 секунды, а пройденный за время разгона путь вновь будет равен 0,6 м. Участок пути, предшествующий разгону, будет пройден за время 1,4 м/(0,2 м/с) = 7 секунд. Полное время движения вдоль отрезка при этом составит t = 9 секунд, а среднее ускорение будет равно aср = v/t 0,0222 м/с2.

Итак, величина среднего ускорения может лежать в пределах от amin 0,0222 м/с2 до amax 0,0364 м/с2. Ничего более определенного сказать нельзя.

9.2. Обозначим силу натяжения нити над грузом 2m через Т.

Тогда сила натяжения нити над грузом m тоже равна Т. Силу натяжения нити снизу от груза m обозначим через Р. Тогда сила натяжения нити над грузом 5m будет равна 2Р. Обозначим ускорение груза m через а и направим координатную ось X вниз. В этом случае ускорение груза 2m равно а. Ускорение же груза 5m, прикрепленного к оси подвижного блока, совпадает по направлению с ускорением груза m и меньше него в 2 раза, то есть равно а/2. Теперь можно записать уравнения динамики для каждого из тел:

Отсюда a = 6g/17.

9.3. Будем считать, что теплоотдача в окружающую среду при изменении температуры в пределах от +55 C до +56 C остается неизменной и обозначим мощность тепловых потерь через Р. Тогда из условия задачи следует, что мощность кипятильника N за вычетом мощности тепловых потерь втрое превышает мощность тепловых потерь, то есть N Р = 3Р. Отсюда N = 4Р. Так как половина мощности кипятильника превышает мощность тепловых потерь, то после падения напряжения в сети вода все равно будет нагреваться. При упавшей мощности кипятильника время нагревания воды на 1 градус должно составить 3 минуты, то есть на 0,5 градуса вода нагреется за 1,5 минуты.

Заметим, что значение температуры в комнате в ответ не входит оно нужно только для того, чтобы можно было обосновать постоянство теплоотдачи при изменении температуры в пределах от +55 C до +56 C.

9.4. Из условия задачи следует, что при подключении вольтметра параллельно резистору падение напряжения на другом резисторе в 2 раза превышает падение напряжения на вольтметре с резистором.

Это означает, что сопротивление параллельно соединенных вольтметра и резистора ровно вдвое меньше, чем у второго (такого же) резистора. Поэтому сопротивления резистора и вольтметра одинаковы. Если соединить их последовательно и подключить к батарейке, то напряжение батарейки поделится между резистором и вольтметром пополам, и вольтметр покажет напряжение 3 В.

10.1. Нити образуют ромб, горизонтальная диагональ которого распорка. Обозначим углы, образованные участками нитей с вертикалью, через. Тогда sin = 0,5d/(0,5L) = d/L. Силы натяжения участков нити Т одинаковы (если бы распорка имела массу, то силы натяжения верхних участков нити были бы больше). Сумма вертикальных проекций двух сил натяжения удерживает груз массой m в покое. Сумма горизонтальных проекций сил натяжения двух нитей давит на распорку именно эту силу и нужно найти, поскольку вторая половинка распорки давит на первую и компенсирует упомянутую силу. Условия равновесия груза и распорки в проекциях на вертикальное и горизонтальное направления, соответственно, имеют вид: 2T cos = mg, 10.2. Пусть начальная скорость налетающего кубика равна v, а его масса равна m. Тогда, в соответствии с законом сохранения импульса, после удара скорость мягкого кубика будет равна v/5, а скорость твердого кубика составит 4v/5. Теперь можно, используя закон изменения механической энергии, выразить количество теплоты Q, выдеmv 2 m(v/5)2 m(4v/5) Энергия деформации максимальна тогда, когда скорости кубиков одинаковы и равны v/2. Их суммарная кинетическая энергия в этот момент составляет mv 2 /4. К этому моменту часть начального запаса кинетической энергии уже перешла в тепло, а часть превратилась в потенциальную энергию деформации Wдеф. К рассматриваемому моменту выделилась половина от найденного нами общего количества теплоты, так как теплота выделяется как при сжатии, так и при растяжении мягкого кубика. Тогда закон изменения механической энергии можно записать в виде: mv 2 /2 = mv 2 /4 + Q/2 + Wдеф. Отсюда Wдеф = 17mv 2 /100, то есть Q/Wдеф = 16/17.

10.3. В соответствии с законом Авогадро один моль идеального газа при нормальных условиях занимает объем V0 = 22,4 л. Поэтому масса одного моля смеси водорода и азота равна М = V0 = 4,48 г.

Пусть масса водорода в = 1 моле смеси равна m, тогда азот в этой порции смеси газов имеет массу M m. Следовательно, масс азота и водорода составляет (M m)/m 1,5.

10.4. Будем считать, что теплоотдача в окружающую среду при изменении температуры в пределах от +55 C до +56 C остается неизменной, и обозначим мощность тепловых потерь через Р. Тогда из условия задачи следует, что мощность кипятильника N за вычетом мощности тепловых потерь вдвое превышает мощность тепловых потерь, то есть N Р = 2Р. Отсюда N = 3P. Поэтому для поддержания постоянной температуры воды +55 C нужно снизить мощность кипятильника в 3 раза.

Теперь попытаемся ответить на второй вопрос задачи. Если считать, что теплоотдача стакана с водой в окружающую среду пропорциональна разности температур стакана и комнаты, то при помощи данного кипятильника можно было бы нагреть воду до утроенной разности температур: (55 22) · 3 = 99 градусов. Это дало бы конечную температуру стакана +121 C, что выше температуры кипения. Таким образом, мощности данного кипятильника в принципе должно хватать для того, чтобы довести воду в стакане до кипения. Однако, следует помнить, что при повышении температуры сильно возрастает интенсивность испарения, что приводит к резкому возрастанию затрат энергии на нагревание.

Следовательно, ответ на второй вопрос задачи неоднозначен: если стакан ничем не прикрыт сверху для уменьшения испарения, то мощности кипятильника может и не хватить для доведения воды до кипения.

10.5. Обозначим силу тока в цепи при последовательном соединении приборов через I. Из условия задачи следует, что сопротивления вольтметров относятся как 0,5 : 1 : 2, а напряжение батарейки составляет 0,5 В + 1 В + 2 В = 3,5 В. При изменении схемы соединения сила тока в цепи батарейки изменилась, поэтому остаться прежними могли только показания одного из параллельно соединенных приборов. Проанализируем возможные варианты.

Пусть параллельно соединены вольтметры 0,5 В и 1 В, и показания первого из них остались равными 0,5 В. Второй вольтметр при этом также должен показывать 0,5 В. Сила тока, текущего через первый вольтметр, равна I, а через второй, имеющий вдвое большее сопротивление 0,5I. Через третий вольтметр течет суммарный ток 1,5I, и поэтому он показывает напряжение 2 · 1,5 = 3 В. В сумме напряжение 0,5 В, падающее на двух параллельно соединенных вольтметрах, и напряжение 3 В, падающее на третьем вольтметре, дают как раз 3,5 В, то есть напряжение батарейки. Итак, рассмотренный вариант подходит.

Осталось проверить, нет ли других возможных вариантов.

Пусть теперь параллельно соединены вольтметры 2 В и 1 В, и показания первого из них остались равными 2 В. Тогда второй вольтметр тоже будет показывать 2 В. Сила тока, текущего через первый вольтметр, будет равна I, а сила тока, текущего через второй вольтметр, будет равна 2I. При этом через третий вольтметр будет течь ток силой 3I, а падающее на нем напряжение составит 3 · 0,5 = 1,5 В. Суммарное падение напряжения в цепи при этом составит 3,5 В, что соответствует напряжению батарейки. Таким образом, и этот вариант соединения подходит.

Пусть параллельно соединены вольтметры 1 В и 2 В, и показания первого из них остались равными 1 В. Значит, и второй вольтметр будет показывать 1 В. Через первый вольтметр при этом течет ток I, а через второй ток I/2. Через третий вольтметр течет суммарный ток 1,5I, и этот вольтметр показывает напряжение 1,5 · 0,5 = = 0,75 В. Следовательно, суммарное падение напряжения в цепи составляет 1 В+0,75 В = 1,75 В, что не соответствует напряжению батарейки.

Следовательно, этот вариант соединения не подходит.

Рассуждая аналогично, можно проверить, что не подходят следующие варианты соединения: с параллельным подключением вольтметров 1 В и 0,5 В при условии сохранения неизменными показаний вольтметра 1 В ; с параллельным соединением вольтметров 2 В и 0,5 В при условии сохранения показаний первого вольтметра 2 В ;

с параллельным соединением вольтметров 0,5 В и 2 В при условии сохранения показаний первого вольтметра 0,5 В.

Итак, возможны два варианта соединения вольтметров, удовлетворяющие условию задачи: 1) параллельно соединены вольтметры 0,5 В и 1 В, и показания первого из них остались равными 0,5 В;

2) параллельно соединены вольтметры 2 В и 1 В, и показания первого из них остались равными 2 В.

Московская региональная олимпиада школьников по физике, (IV этап Всероссийской олимпиады школьников) 9.1. Представим себе, что корабли проплыли друг мимо друга и после момента их встречи прошел один час. Тогда расстояние между кораблями будет составлять 40 морских миль, а шлейфы их дымов пересекутся в некоторой точке, которая будет находиться западнее первого корабля (см. рис.). Заметим, что в месте пересечения шлейфов находятся клубы дыма, которые были испущены в момент встречи кораблей. Так как скорости кораблей одинаковы, то точка встречи находится посередине между кораблями, то есть на расстоянии 20 морских миль от первого корабля. Таким образом, за 1 час клубы дыма под действием ветра передвинулись на расстояние s = 402 + 202 = 2000 44,7 миль, то есть скорость ветра равна V 44,7 узла 82,8 км/ч. Из рисунка видно, что направление ветра составляет с курсом первого корабля угол, определяемый из соотношемиль ния tg = 9.2. Обозначим радиус дуги через R, силы реакции, действующие на проволоку со стороны верхнего и нижнего гвоздей, через N и F соответственно, а расстояние OD от центра окружности, частью которой является дуга, до центра масс проволочной фигуры через h (см. рисунок).

Отметим, что центр масс фигуры лежит на оси ее симметрии.

Так как проволока гладкая, то сила F направлена к центру указанной окружности. Введем координатные оси X (горизонтальную) и Y (вертикальную) и запишем условия равновесия проволоки:

Здесь Nx и Ny проекции силы N на координатные оси, = /3 угол между вертикалью и диаметром AC при наличии нижнего гвоздя. При отсутствии нижнего гвоздя, когда проволока находится в положении равновесия, ее центр масс лежит на вертикальной прямой, проходящей через верхний гвоздь. Следовательно, h = R tg 0. Решая с учетом этого полученную систему уравнений, имеем Искомая сила T, с которой проволока давит на гвоздь, равна по величине силе N и противоположна ей по направлению. Таким образом, Заметим, что заданный в условии задачи угол 0 можно найти при помощи следующего приема. Расположим проволочную дугу так, чтобы ее концы лежали на одной вертикальной прямой, после чего повернем ее на малый угол вокруг оси, проходящей перпендикулярно плоскости дуги через точку O. При этом потенциальная энергия дуги, с одной стороны, увеличится на m · · g · 2R, так как элемент проволоки массой m · поднимется на высоту 2R. С другой стороны, это же увеличение потенциальной энергии равно mg · h, так как центр масс дуги поднимется на высоту h. Приравнивая эти выражения, получим, что h = 2R/, а tg 0 = h/R = 2/. Поэтому полученный выше ответ задачи можно не исследовать содержащаяся под корнем величина заведомо положительна.

9.3. Поскольку нить и блоки невесомые и трения нет, то на левый груз в горизонтальном направлении действует сила (см. рис.), равная 3F и направленная слева направо, а на правый груз сила 2F, направленная справа налево. Направим ось X неподвижной системы координат направо. Тогда проекция ускорения левого груза на ось X будет, очевидно, равна a1 = 3F/M, а проекция ускорения правого груза a2 = 2F/M.

Найдем, как связаны друг с другом ускорения грузов и конца нити, то есть получим уравнение кинематической связи. Для этого обозначим координату оси левого блока в некоторый момент времени t через x1 (t), координату оси правого блока через x2 (t), а координату конца нити через x(t). Пусть длина нити равна L, радиусы блоков r, расстояние от оси левого блока до левого груза x0. Так как нить нерастяжима, то можно выразить ее постоянную длину L через введенные координаты:

Отсюда Такое же соотношение справедливо также и для момента времени t + t, близкого к моменту t:

Вычитая из второго соотношения первое, найдем связь между перемещениями левого и правого грузов x1 и x2 и смещением конца нити x: x = 3x1 2x2. Деля полученное уравнение на величину промежутка времени t, найдем связь между скоростями грузов и конца нити: v = 3v1 2v2. Это соотношение также справедливо для любых двух близких моментов времени. Поэтому связь между ускорениями грузов может быть найдена аналогично и имеет вид: a = 3a1 2a2. Следует отметить, что ускорение правого груза a2 направлено влево, против оси X, и поэтому отрицательно.

Таким образом, из нерастяжимости нити следует, что ускорение, с которым движется конец нити, складывается из утроенной величины ускорения левого груза и удвоенной величины ускорения правого груза:

9.4. Пусть сопротивление переменного резистора R = 0. Тогда схему можно перерисовать в виде, показанном на рисунке 1. Полное сопротивление такой цепи равно r1 = 2R1 R2 /(R1 + R2 ), а текущий через миллиамперметр ток равен Если сопротивление переменного резистора, напротив, очень велико, то схему можно перерисовать так, как показано на рисунке 2.

Сопротивление этой цепи равно r2 =, а ток, текущий через миллиамперметр, равен Исключая из полученной системы уравнений, например, величину R2, приходим к квадратному уравнению, позволяющему определить R1 :

Отсюда Заметим, что формулы получились симметричными выражения для R1 и R2 переходят друг в друга при замене знака перед квадратным корнем. Это связано с тем, что исходная схема включения резисторов также симметрична.

Из графика, приведенного в условии на правом рисунке, видно, что I1 = 1 · 103 А, I2 = 0,75 · 103 А. Подставляя эти значения в полученные формулы и выбирая в первой перед корнем знак +, а во второй знак, найдем: R1 = 6 кОм, R2 = 2 кОм. При противоположном выборе знаков получится R1 = 2 кОм, R2 = 6 кОм.

10.1. Перейдем в систему отсчета, связанную с движущимся цилиндром. В ней перед ударом шарик движется со скоростью v + u (см. рис.).

Так как цилиндр гладкий и между ним и шариком не действуют силы трения, то углы между нормалью к поверхности цилиндра в месте удара и направлениями движения шарика до и после удара одинаковы.

Следовательно, после удара о цилиндр шарик в движущейся системе отсчета будет двигаться с той же по модулю скоростью v + u под углом = 2 к направлению исходного движения, причем cos = L/R.

Вернемся обратно в неподвижную систему отсчета, связанную с землей. В ней скорость шарика после удара v1 будет представлять собой векторную сумму скорости u цилиндра относительно земли и скорости v + u шарика относительно цилиндра (см. рис.). Используя теорему косинусов, получим:

С учетом выражения для cos окончательно получаем:

10.2. Скорость шайбы v на расстоянии S от точки начала движения можно определить из закона изменения механической энергии.

При начальной скорости v0 и массе шайбы m имеем:

где A(S) работа, которую совершает сила трения на пути S.

При достаточно малом перемещении S справедлива формула A(S) = µ(S)mgS, где µ(S) коэффициент трения на расстоянии S от точки начала движения. Видно, что величина A(S) не зависит от скорости v0, а значит, и суммарная работа A(S) на всем пути S не зависит от v0, пока v(S) 0. Поэтому при начальной скорости шайбы v1 она остановится, пройдя путь S, который можно определить из соотношения 1 = A(S). Подставляя A(S) из этого соотношения в предыдущее уравнение, получаем:

откуда находим величину скорости v на нашем графике, соответствующую точке остановки шайбы во втором случае: v = v0 v1 = 3 м/с.

Из графика находим, что скорости v = 3 м/с соответствует S = = 5 м, то есть при начальной скорости v1 = 4 м/с шайба пройдет до полной остановки путь 5 м.

10.3. Поскольку давление в точке A меньше давления насыщенных паров при температуре TA = 300 К, то все вещество находится в газообразном состоянии. Если бы вещество не конденсировалось, то давление p и температура T этого вещества в соответствии с уравнением Менделеева Клапейрона были бы связаны прямо пропорциональной зависимостью p = T. В условиях, когда вещество из пара переходит в конденсированное состояние, линейная зависимость давления от температуры при понижении температуры справедлива только до момента начала конденсации, который соответствует точке пересечения графика, приведенного на рисунке, с прямой линией p = T, проходящей через начало координат и точку A.

Если, как это требуется по условию задачи, сконденсировалась половина имеющегося вещества, то это состояние будет соответствовать некоторой точке B на кривой зависимости давления насыщенного пара от температуры. Эту точку можно найти путем следующих рассуждений. Если все сконденсировавшееся вещество удалить, то давление в системе останется равным давлению насыщенного пара в точке B. Если теперь нагревать оставшийся пар до начальной температуры TA = 300 К, то его давление, очевидно, будет линейно возрастать до величины вдвое меньшей, чем было начальное давление pн, поскольку количество вещества в системе уменьшилось вдвое. Отсюда следует графический способ нахождения температуры, соответствующей точке B.

Поставим на pT –диаграмме вспомогательную точку C, соответствующую начальной температуре TA = 300 К и давлению pн /2. Проведем через точку C и начало координат прямую. Эта прямая пересечет график зависимости давления насыщенного пара от температуры в искомой точке B, соответствующей в нашем случае температуре T 200 К.

10.4. Так как конденсатор разряжается не до конца, то находящиеся в черном ящике конденсатор и резистор могут быть соединены друг с другом и с контактами только последовательно. Поскольку установившийся заряд на конденсаторе C равен Q0 /2, то емкость конденсатора C1 в черном ящике также равна C.

В начальный момент времени ток через резистор, в соответствии с определением силы тока, равен I =, где Q заряд, стекший с конденсатора C за малое время t, прошедшее после начала его разрядQ ки. Отношение равно угловому коэффициенту касательной к граt фику зависимости Q(t), проведенной в точке, соответствующей моменту времени t = 0. Из графика (см. рис.) видно, что эта касательная, выходящая из точки Q0 на оси ординат, отсекает на оси абсцисс отрезок времени, равный. Таким образом, начальный ток через резистор сопротивление резистора R = =.

10.5. Из приведенного в условии графика ясно, что стенки баллона имеют толщину 1 см, а внешний диаметр баллона d = 4 см. Для того, чтобы получить ответ, достаточно сравнить интенсивность излучения на середине баллона при x = 2 см (в этом месте луч проходит через содержимое баллона и через две его стенки, суммарная толщина которых 2 см) и там, где при проходе сквозь стенку толщина металла на пути луча также равна 2 см. Если они одинаковы, то баллон пуст.

Из геометрических соображений (см. рис.) следует:

где r = d/2 = 2 см, h = 1 см. Отсюда Из графика видно, что при x = 2 см интенсивность излучения составляет около 0,4 ед., а при x = x1 и при x = x2 интенсивность равна приблизительно 0,5 ед., то есть несколько больше. Значит, в баллоне есть содержимое, поглощающее рентгеновские лучи.

Федеральный окружной (IV) этап Всероссийской олимпиады 9.1. См. рисунок.

µgt 9.4 Цифры 0, 1, 2, 3 показывают количество изображений, наблюдаемых в каждой из областей на рисунке.

10.1. Ускорение равно a = вертикально вниз.

Q31 + Q1K Заключительный (V) этап Всероссийской олимпиады 9.3. Время одинаково для обоих случаев и равно = м + с = = 63 мин.

9.4. В резисторе R5 сила тока минимальна и равна Imin = 2 мА, а в резисторе R4 максимальна и равна Imax = 13,3 мА.

10.1. 1) vCотн = 0,5v = 10 км/ч; 2) vDотн = 5,5v = 110 км/ч.

10.5. Q = 25 Дж.

7. Программа Всероссийской олимпиады Программа 9 класса Измерение физических величин. Погрешность измерения. Построение графика по результатам эксперимента, выбор переменных.

Механическое движение. Относительность движения. Система отсчета. Координаты. Материальная точка. Траектория. Путь и перемещение. Средняя и мгновенная скорость. Ускорение. Прямолинейное движение. Свободное падение.

Движение по окружности. Частота обращения. Угловая скорость.

Центростремительное ускорение.

Механические колебания. Амплитуда, период, частота колебаний.

Механические волны. Длина волны. Звук.

Взаимодействие тел. Трение. Упругая деформация. Инерция.

Масса. Импульс. Первый закон Ньютона. Инерциальная система отсчета. Сила. Принцип суперпозиции сил. Второй закон Ньютона. Силы в природе: сила тяготения, сила тяжести, сила трения, сила упругости.

Закон всемирного тяготения. Искусственные спутники Земли. Третий закон Ньютона. Закон сохранения импульса. Ракеты.

Работа. Мощность. Кинетическая энергия. Потенциальная энергия. Закон сохранения механической энергии.

Элементы статики. Момент силы. Условие равновесия твердого тела.

Давление. Атмосферное давление. Гидростатическое давление.

Передача давления твердыми телами, жидкостями и газами. Закон Паскаля. Закон Архимеда. Гидравлический пресс. Уравнение Бернулли.

Применение законов Ньютона и законов сохранения импульса и энергии для анализа и расчета движения тел. Простые механизмы.

КПД механизмов.

Методы исследования механических явлений Измерительные приборы: измерительная линейка, штангенциркуль, часы, мерный цилиндр, динамометр, барометр. Измерение расстояний, промежутков времени, силы, объема, массы, давления. Графики изменения со временем кинематических величин.

Молекулярная физика. Термодинамика Гипотеза о дискретном строении вещества. Непрерывность и хаотичность движения частиц вещества. Диффузия. Броуновское движение. Модели газа, жидкости и твердого тела. Плотность. Взаимодействие частиц вещества.

Внутренняя энергия. Температура. Термометр. Теплопередача.

Необратимость процесса теплопередачи. Связь температуры с хаотическим движением частиц. Тепловое расширение твердых тел и жидкостей. Температурные коэффициенты линейного и объемного расширения. Особенности теплового расширения воды. Количество теплоты. Удельная теплоемкость. Удельная теплота сгорания топлива. Закон сохранения энергии в тепловых процессах.

Испарение жидкости. Удельная теплота парообразования (конденсации). Влажность воздуха. Кипение жидкости. Плавление твердых тел.

Применение основных положений молекулярно-кинетической теории вещества для объяснения разной сжимаемости твердого тела, жидкости и газа; процессов испарения и плавления; преобразования энергии при плавлении и испарении вещества. Удельная теплота плавления (кристаллизации).

Преобразования энергии в тепловых двигателях.

Методы исследования тепловых явлений Измерительные приборы: термометр, манометр, гигрометр.

Измерение температуры, давления газа, влажности воздуха. Графики изменения температуры вещества при его нагревании и охлаждении, кипении и плавлении.

Электродинамика Электризация тел. Электрический заряд. Взаимодействие зарядов. Два вида электрического заряда. Закон сохранения электрического заряда. Электрическое поле. Действие электрического поля на электрические заряды.

Постоянный электрический ток. Сила тока. Напряжение. Электрическое сопротивление. Электрическая цепь. Закон Ома для участка цепи. Преобразование энергии при нагревании проводника с электрическим током. Носители электрических зарядов в различных средах.

Взаимодействие магнитов. Магнитное поле. Взаимодействие проводников с током. Действие магнитного поля на электрические заряды.

Электродвигатель.

Электромагнитная индукция. Преобразование энергии в электрогенераторах.

Электромагнитные волны. Скорость распространения электромагнитных волн. Равенство скоростей электромагнитной волны и света. Свет электромагнитные волны. Прямолинейное распространение.

Луч. Отражение и преломление света. Закон отражения света. Плоское зеркало. Закон преломления света. Линза. Фокусное расстояние.

Методы исследования электромагнитных явлений Измерительные приборы: амперметр, вольтметр, счетчик электрической энергии. Измерение силы тока, напряжения, сопротивления проводника. Расчет простейшей электрической цепи. Построение изображения в плоском зеркале, собирающей и рассеивающей линзе. Оптические приборы.

Атомная физика Опыты Резерфорда. Планетарная модель атома. Радиоактивность. Альфа-, бета- и гамма-излучения. Атомное ядро. Протоннонейтронная модель ядра. Зарядовое и массовое число. Изотопы.

Ядерные реакции. Деление и синтез ядер. Сохранение заряда и массового числа при ядерных реакциях. Применение законов сохранения для расчета простейших ядерных реакций. Энергия связи частиц в ядре. Выделение энергии при делении и синтезе ядер. Излучение звезд.

Ядерная энергетика.

Методы наблюдения и регистрации частиц в ядерной физике.

Дозиметрия.

Программа 10 класса Механика Принцип относительности Галилея.

Закон всемирного тяготения. Движение под действием силы тяготения. Первая космическая скорость. Невесомость.

Сила упругости. Закон Гука. Закон сохранения импульса. Работа силы. Кинетическая энергия. Потенциальная энергия. Упругий и неупругий удар.

Математический маятник. Гармонические колебания. Амплитуда, частота, период, фаза колебаний. Свободные колебания. Вынужденные колебания. Автоколебания. Резонанс. Волны. Длина волны. Скорость распространения волны. Уравнение гармонической волны.

Молекулярная физика. Термодинамика Основы молекулярной физики. Экспериментальные основания молекулярно-кинетической теории. Опыты Штерна и Перрена. Масса и размеры молекул. Количество вещества. Моль. Постоянная Авогадро.

Термодинамика. Тепловое равновесие. Температура. Связь температуры со средней кинетической энергией частиц вещества. Постоянная Больцмана. Абсолютный нуль. Количество теплоты. Теплоемкость.

Первый закон термодинамики. Адиабатический процесс. Второй закон термодинамики и его статистическое истолкование. Тепловые машины.

КПД теплового двигателя.

Идеальный газ. Давление газа. Связь между давлением и средней кинетической энергией молекул идеального газа. Уравнение Менделеева–Клапейрона. Работа при изменении объема идеального газа. Изопроцессы.

Жидкость и твердое тело. Относительная влажность. Кипение.

Зависимость температуры кипения жидкости от давления. Насыщенный и ненасыщенный пар. Зависимость давления насыщенного пара от температуры. Психрометр. Гигрометр. Кристаллические и аморфные тела. Поверхностное натяжение. Смачивание. Капиллярные явления.

Деформация.

Электростатика Электрический заряд. Элементарный заряд. Закон Кулона. Электрическое поле. Напряженность электрического поля. Потенциальность электрического поля. Разность потенциалов. Принцип суперпозиции полей. Проводники в электрическом поле. Электрическая емкость. Конденсатор. Диэлектрики в электрическом поле. Поляризация диэлектриков. Энергия электрического поля конденсатора. Плотность энергии.

Постоянный электрический ток Электрический ток. Носители свободных электрических зарядов в металлах, жидкостях и газах. Сила тока. Работа тока. Напряжение.

Мощность тока. Электродвижущая сила. Закон Ома для полной электрической цепи. Сопротивление при последовательном и параллельном соединении проводников. Шунты и добавочные сопротивления. Правила Кирхгофа.

Полупроводники. Собственная и примесная проводимость полупроводников, pn – переход.

Программа 11 класса В программу 11 класса включается весь материал программ 9 и 10 классов, а также следующие темы.

Магнитное поле Индукция магнитного поля. Сила Ампера. Сила Лоренца. Магнитный поток.

Электромагнитное поле. Закон электромагнитной индукции Фарадея. Вихревое электрическое поле. Самоиндукция. Индуктивность. Электромагнитные колебания в колебательном контуре.

Переменный ток. Производство, передача и потребление электрической энергии.

Идеи теории Максвелла. Электромагнитное поле. Электромагнитные волны. Свойства электромагнитных волн. Радио. Телевидение.

Волновые свойства света Свет электромагнитные волны. Скорость света и методы ее измерения. Интерференция света. Когерентность. Дифракция света.

Дифракционная решетка. Поляризация света. Закон преломления света. Призма. Дисперсия света. Формула тонкой линзы. Получение изображения при помощи тонкой линзы. Оптическая сила линзы. Глаз.

Очки. Лупа. Микроскоп. Зрительная труба. Фотоаппарат. Проекционный аппарат.

Основы специальной теории относительности Инвариантность скорости света. Принцип относительности. Пространство и время в специальной теории относительности. Релятивистский закон сложения скоростей. Закон взаимосвязи массы и энергии.

Квантовая физика Тепловое излучение. Постоянная Планка. Фотоэффект. Опыты Столетова. Фотоны. Опыты Вавилова. Уравнение Эйнштейна для фотоэффекта. Корпускулярно-волновой дуализм.

Гипотеза Луи де Бройля. Дифракция электронов. Боровская модель атома водорода. Спектры. Люминесценция. Лазеры.

Закон радиоактивного распада. Нуклонная модель ядра. Энергия связи нуклонов в ядре. Деление ядра. Синтез ядра. Ядерная энергетика.

Элементарные частицы. Фундаментальные взаимодействия.



Pages:     || 2 |


Похожие работы:

«Введение В основу настоящей программы положены следующие дисциплины: мониторинг среды обитания человека, механика сплошных сред, управление в технических системах, теория электромеханических процессов, тепло- и массоперенос в системах жизнеобеспечения, теория надежности и эффективности, системотехника, теория проектирования систем жизнеобеспечения летательных аппаратов, имитационное и математическое моделирование. Раздел 1. Внешние условия жизнедеятельности 1.1. Человек - система - среда...»

«2634 Министерство транспорта РФ ФЕДЕРАЛЬНОЕ АГЕНТСТВО ЖЕЛЕЗНОДОРОЖНОГО ТРАНСПОРТА ГОСУДАРСТВЕННОЕ ОБРАЗОВАТЕЛЬНОЕ УЧРЕЖДЕНИЕ ВЫСШЕГО ПРОФЕССИОНАЛЬНОГО ОБРАЗОВАНИЯ САМАРСКИЙ ГОСУДАРСТВЕННЫЙ УНИВЕРСИТЕТ ПУТЕЙ СООБЩЕНИЯ Кафедра Экономика и логистика на транспорте Формирование бюджета производства в дистанции электроснабжения в соответствии с финансово-экономической моделью ОАО РЖД Методические указания и задание к выполнению контрольной работы по дисциплине Экономика хозяйства электроснабжения для...»

«С.Н.Литвинова Организация досуга детей и подростков (Методическое пособие для педагогов системы дополнительного образования и для родителей) В методическом пособии освещены современные подходы к организации досуговой деятельности в школах, в центрах дополнительного образования по месту жительства. Показана специфика применения методов досуговой деятельности, направленных на воспитание детей и подростков с учетом возрастных особенностей. Охарактеризованы виды досуга и формы организации досуговой...»

«Министерство транспорта и связи Украины Государственный департамент по вопросам связи и информатизации Одесская национальная академия связи им. А.С. Попова Подготовительное отделение для иностранцев Кафедра украинского и русского языков ГЛАГОЛЫ ДВИЖЕНИЯ УЧЕБНОЕ ПОСОБИЕ ДЛЯ СТУДЕНТОВ-ИНОСТРАНЦЕВ Одесса-2008 1 УДК 808. 2 (07) План УМН 2007/2008 уч. год Составители: С.А.Карпова Л.Е.Расходчикова Л.А.Сокольницкая Отв. редактор: Н.А.Маслова Учебное пособие Глаголы движения посвящено одной из наиболее...»

«Негосударственное образовательное учреждение высшего профессионального образования Московский экономико-правовой институт РАБОЧАЯ ПРОГРАММА ДИСЦИПЛИНЫ ФИНАНСОВЫЕ РЫНКИ И ИНСТИТУТЫ образовательная программа направления подготовки 080200.62 Менеджмент Цикл Б.3 Профессиональный, базовая часть Профиль подготовки Финансовый менеджмент Квалификация (степень) выпускника Бакалавр Москва 2013 2 Рабочая программа дисциплины Финансовые рынки и институты составлена в соответствии с требованиями...»

«Методические рекомендации по обучению русскому языку и литературе на профильном уровне в современных условиях Владение русским языком, умение общаться, добиваться успеха в процессе коммуникации являются теми характеристиками личности, которые во многом определяют достижения выпускника школы практически во всех областях жизни, способствуют его социальной адаптации к изменяющимся условиям современного мира. В процессе изучения русского языка создаются предпосылки для восприятия и понимания...»

«Геоморфология И.С. Щукина и других советских ученых Иван Степанович Щукин (1885- 1985) - организатор и первый заведующий кафедрой геоморфологии в МГУ, один из очень немногих географов, отметивший 100-летний юбилей. Университет Щукин закончил за три года уже в зрелом возрасте и был оставлен на кафедре Д.Н. Анучина. С 1918 г. начал читать самостоятельные курсы лекций по географии и народоведению Кавказа и одновременно преподавать географию в Межевом институте (теперь Институт геодезии,...»

«Министерство образования Республики Беларусь Учреждение образования ГРОДНЕНСКИЙ ГОСУДАРСТВЕННЫЙ УНИВЕРСИТЕТ ИМЕНИ ЯНКИ КУПАЛЫ МЕТОДИЧЕСКИЕ УКАЗАНИЯ ПО ВЫПОЛНЕНИЮ КУРСОВЫХ И ДИПЛОМНЫХ РАБОТ для студентов специальностей 1-02 03 06-01 – Английский язык, 1-02 03 06-02 – Немецкий язык, 1-02 03 06-04 – Французский язык, 1-21 06 01-01 – Современные иностранные языки, 1-23 01 02 – Лингвистическое обеспечение межкультурных коммуникаций Гродно 2007 УДК 378 (075) ББК 74.580я73 М54 Составители: Середа...»

«РОССИЙСКАЯ ЭКОНОМИЧЕСКАЯ АКАДЕМИЯ им. Г.В.Плеханова ФИНАНСОВЫЙ БИЗНЕС-ПЛАН Под редакцией действительного члена Ажадемин инвестиций Российской Федерации, доктора экономических наук, профессора В.М.Попом Рекомендовано Учебно-методическим объединением Финансовой академии при Правительстве Российской Федерации в качестве учебного пособия для студентов экономических специальностей Москва Финансы и статистика 2002 УДК 658.012.2 ББК 65.290-2 Ф59 А вторский коллектив: В.М. Попов (руководитель), С.И....»

«МИНИСТЕРСТВО ОБРАЗОВАНИЯ РОССИЙСКОЙ ФЕДЕРАЦИИ Санкт-Петербургская государственная лесотехническая академия В. Н. ПЕТРОВ ЭКОНОМИКА НЕДВИЖИМОСТИ УЧЕБНОЕ ПОСОБИЕ Допущено Учебно-методическим объединением по образованию в области производственного менеджмента в качестве учебного пособия для студентов, обучающихся по специальности 060800 Экономика и управление на предприятии лесного хозяйства, деревообрабатывающей и целлюлозно-бумажной промышленности. САНКТ-ПЕТЕРБУРГ НАУКА 2003 УДК 33:630*91:332.2...»

«ГБСК ОУ школа №663 Московского района Санкт-Петербурга Конспект урока русского языка в 5 классе на тему Употребление имен существительных множественного числа в речи учителя русского языка и литературы ГБСК ОУ школы №663 Московского района Санкт-Петербурга Кашиной Надежды Ивановны Директор школы (Семенова А.А.) 11.04.2012 г. Санкт-Петербург 2012 г. Программа: Баранов М.Т., Ладыженская Т.А., Шанский Н.М. Программа по русскому языку к учебникам для 5-9 классов, М., Просвещение, 2000 г. Учебник...»

«Государственное образовательное учреждение высшего профессионального образования Дагестанский государственный педагогический университет Детский Фонд ООН (ЮНИСЕФ) Методика преподавания прав ребенка Учебно-методическое пособие для студентов высших учебных заведений Махачкала 2008 www.unicef.ru УДК 347.631 ББК 74.263.8 М 19 Печатается по решению Научно-экспертного совета и Учебно-методического объединения ГОУ ВПО Дагестанский государственный педагогический университет Авторский коллектив: Д.М....»

«БИБЛИОГРАФИЧЕСКИЙ УКАЗАТЕЛЬ КНИГ, ПОСТУПИВШИХ В БИБЛИОТЕКУ (декабрь 2012 г.) БИОЛОГИЯ БИОХИМИЯ 1. 577.1(075) Б 63 Биоорганическая химия : Лабораторный практикум / сост. Г. Е. Ванина, И. В. Рашевская. - Пенза : Изд-во Пенз. гос. пед. ун-та, 2012. - 108 с. Экземпляры: всего:2 - чз6(2) БОТАНИКА 2. 58(075) С 30 Семёнова, Елена Фёдоровна Практикум по ботанике : учебное пособие / Е. Ф. Семёнова, Н. А. Меженная, Т. М. Фадеева. - Пенза : Изд-во Пенз. гос. ун-та, 2012. - 1672 с. : ил. Экземпляры:...»

«Министерство образования и науки РФ Федеральное государственное бюджетное образовательное учреждение высшего профессионального образования Уральский государственный педагогический университет Институт иностранных языков ВЫПУСКНАЯ КВАЛИФИКАЦИОННАЯ РАБОТА МЕТОДИЧЕСКИЕ РЕКОМЕНДАЦИИ ПО ПОДГОТОВКЕ, ОФОРМЛЕНИЮ И ЗАЩИТЕ для специальностей 050303 – Иностранный язык 031202 – Перевод и переводоведение IFL Екатеринбург 2012 УДК 378.147.88 (075.8) ББК Ч 481.268 В 92 Рецензенты: д.ф.н., проф. Г.Н. Бабич,...»

«Рабочая программа по биологии. 8 класс. Базовый уровень. РАБОЧАЯ ПРОГРАММА по биологии, 8 класс 2 часа в неделю, 68 часов. Учебник В.В. Пасечник, А.А. Каменский. Биология, 8 класс. М.: Просвещение,2012г. Программа: Биология. 5-11 классы: программы для общеобразовательных учреждений к комплекту учебников, созданных под руководством В. В. Пасечника/авт.-сост. Г. М. Пальдяева.-2-е изд., стереотип.-М.: Дрофа, 2010. Методическое обеспечение программы 1. Н.В.Дубинина, В.В.Пасечник. Тематическое и...»

«2 Содержание ВВЕДЕНИЕ. ОБЩАЯ ХАРАКТЕРИСТИКА БИРСКОГО ФИЛИАЛА ФГБОУ ВПО БАШКИРСКИЙ ГОСУНИВЕРСИТЕТ 1 1. СТРУКТУРА ПОДГОТОВКИ СПЕЦИАЛИСТОВ. 6 1.1.Общие сведения по УГС 050000 – Образование и педагогика в Бф БашГУ 6 1.2. Сведения по специальностям УГС 050000 – Образование и педагогика 8 2.ОРГАНИЗАЦИОННО-ПРАВОВОЕ ОБЕСПЕЧЕНИЕ ОБРАЗОВАТЕЛЬНОЙ ДЕЯТЕЛЬНОСТИ 14 3.СОДЕРЖАНИЕ ПОДГОТОВКИ СПЕЦИАЛИСТОВ 17 3.1.Учебный план 3.2.Учебные программы дисциплин и практик, диагностические средства 3.3.Программы и...»

«ГАОУ МО СПО МПК ОТЧЁТ ПО РЕЗУЛЬТАТАМ САМООБСЛЕДО А. Содержание Цели и задачи самообследования 1. Общие сведения о колледже 2. Организационно-правовое обеспечение образовательного процесса 3. Структура подготовки специалистов 4. Структура и содержание профессиональных образовательных программ 5. Информационно-методическое обеспечение образовательного процесса 6. Организация учебного процесса 7. Воспитательная работа и социальная защита студентов 8. Инновационная деятельность 9. Кадровое...»

«Министерство путей сообщения Российской Федерации Дальневосточный государственный университет путей сообщения Кафедра “Строительные и путевые машины” Г.В. Завгородний СОДЕРЖАНИЕ И РЕМОНТ ЖЕЛЕЗНОДОРОЖНОГО ПУТИ. ПУТЕВЫЕ МАШИНЫ И МЕХАНИЗМЫ Учебно-методическое пособие к выполнению курсовой работы для студентов 1-го курса специальности Подъемно-транспортные, строительные, дорожные машины и оборудование всех форм обучения Хабаровск 2000 УДК ББК Завгородний Г.В. Содержание и ремонт железнодорожного...»

«ГБУЗ КО Кемеровская областная научная медицинская библиотека Научная библиотека ГОУ ВПО КемГМА Росздрава ГУК Кемеровская областная научная библиотека им. В.Д. Федорова Медицинская литература (текущий указатель литературы) Вып. 1 Кемерово - 2014 2 Текущий указатель новых поступлений Медицинская литература издается Кемеровской областной научной медицинской библиотекой совместно с научной библиотекой КемГМА, Кемеровской областной научной библиотекой им. В.Д. Федорова. Библиографический указатель...»

«МЕЖДУНАРОДНЫЙ ИНСТИТУТ ЭКОНОМИКИ И ПРАВА НАЦИОНАЛЬНАЯ ЭКОНОМИКА ПРОБЛЕМНО-ТЕМАТИЧЕСКИЙ КОМПЛЕКС Рекомендовано Министерством образования и науки Российской Федерации в качестве учебного пособия для студентов высших учебных заведений МОСКВА 2008 ББК 65.9(2Рос)я73 П39 УДК [338+316.42](470)(075.8) Рецензенты: д-р экон. наук, проф. А.Е. Хачатуров; кафедра мировой и национальной экономики Всероссийской академии внешней торговли Научный руководитель проекта и автор образовательной технологии Ф.Л....»






 
2014 www.av.disus.ru - «Бесплатная электронная библиотека - Авторефераты, Диссертации, Монографии, Программы»

Материалы этого сайта размещены для ознакомления, все права принадлежат их авторам.
Если Вы не согласны с тем, что Ваш материал размещён на этом сайте, пожалуйста, напишите нам, мы в течении 1-2 рабочих дней удалим его.